SlideShare una empresa de Scribd logo
1 de 25
Descargar para leer sin conexión
CURSOS ON-LINE – ESPANHOL – CURSO REGULAR
          PROFESSOR GERARDO SANMARCO


                            Aula 11

OMC, Blocos regionais e TLCs
Hola amigos!. O cenário do comercio internacional se apresenta
extremamente dinâmico. Se, por um lado, os acordos mundiais de
liberalização de comércio via OMC estão estagnados diante da
posição protecionista da União Européia, os blocos regionais
continuam tentando processos de maior integração (Can e
Mercosur).
Em 2006, o processo que vem alcançando maior repercussão é o
da multiplicação de TLCs, tratados bilaterais de liberalização
comercial. Em nosso continente, os Estado Unidos (que parecem
ter desistido provisoriamente da ALCA) o México e o Chile
despontam como os paises que tem aproveitado com mais
intensidade esses processos para estimular o crescimento das
suas economias.
Do ponto de vista da Fazenda Pública, os processos de
liberalização comercial despertam particular preocupação, porque
eles vêm baseados em redução dos impostos de importação, e
portanto em perda de receita tributária.
Considerando que o parecer técnico da perda de receita tributária
nunca será argumento suficiente para contrabalancear os
benefícios de uma possível melhora do crescimento econômico
nacional, o Ministério de Fazenda deve repensar as estratégias
para preparar mini reformas tributárias que salvaguardem as
necessidades estratégicas do Tesouro.
A ESAF vem acompanhando esse cenário mundial, e tem
colocado textos vinculados a ele, em quase todas as provas de
2000 em diante. O concursando precisa acompanhar muito de
perto as notícias de atualidade, e dominar o vocabulário conceitual
a ele relacionado para não ser tomado por surpresa no próximo
concurso. Recomendamos especialmente a leitura permanente de
noticias em espanhol que marcam a evolução de
•       As negociações da OMC (particularmente do G20)
•       A evolução do Nafta
•       A evolução do CAFTA
•       A evolução do Mercosur
•       A evolução da CAN
•       A evolução da União Européia
•       A evolução da Zona Euro
•       Os TLC em negociação ou em vigência próxima.

Nesta aula retomaremos alguns textos de provas recentes da
ESAF, simulados e leituras que confluem para reforçar os
conhecimentos nessa área. Antes de iniciar os estudos
             www.pontodosconcursos.com.br                        1
CURSOS ON-LINE – ESPANHOL – CURSO REGULAR
             PROFESSOR GERARDO SANMARCO
recomendamos revisar alguns conceitos que aparecerão nas
provas, simulados e leituras que completam esta aula:


FICHA DE VOCABULARIO 10
MAPA CONCEITUAL SOBRE
COMERCIO MUNDIAL E TRIBUTAÇÃO ADUANEIRA

OMC = Organização Mundial do Comércio reúne quase 150
paises foi criada para regular o comércio mundial. Tem alguns
órgãos como o Tribunal de Conflitos que emite pareceres sobre
disputas comerciais. Tem como filosofia a redução gradativa dos
obstáculos (tributários e sanitários) que prejudicam o comercio
mundial e agravam a pobreza das nações menos desenvolvidas.
Convoca conferencias mundiais cada poucos anos para tentar
arrancar acordos dos blocos mais protecionistas: Ronda do
Milênio (Seatle 2000), Ronda de Doha (Emirados 2001),
Conferencia de Cancun (2003), Conferencia de Hong Kong
(2005).
Os grandes temas que enfrentam os eixos Norte/Sur são:
1.- Desgravación arancelaria. Os paises do G20 exigem a
diminuição dos aranceles (impostos aduaneiros) que pesam sobre
Os produtos agrícolas exportáveis. Os paises do G8 exigem como
contrapartida que os paises do G20 reduzam o protecionismo
sobre os produtos industrializados e a quebra das patentes da
industrias farmacêuticas.
2.-Ayudas agrícolas . Apoyo interno.-Os paises do G20
exigem o fim dos subsídios (dinheiro a custo zero) que recebem os
agricultores europeus para sobreviver à concorrência das
exportações dos paises do G20.
3.- Subsidios a exportaciones. O G20 e a União Européia
exigem que Estados Unidos, Japão e China, acabem com as
Tradings (exportadoras) estatais que praticam subsídios
mascarados aos exportadores para compensar sua falta de
competitividade no mercado externo.

Outros termos:

Subvenciones. Apoio em dinheiro que agricultores e/ou
exportadores recém do Estado como compensação diante da
concorrência com outros paises. A OMC o considera “dumping” ou
concorrência desleal” e permite aos paises vítimas, o direito de
retaliação.
Salvaguardas o Derechos compensatórios.
Medidas de retaliação ( impostos adicionais ou quotas
alfandegárias) permitidas pela OMC quando um pais é vitima de

             www.pontodosconcursos.com.br                       2
CURSOS ON-LINE – ESPANHOL – CURSO REGULAR
              PROFESSOR GERARDO SANMARCO
“dumping” por parte das exportações de outro. As medidas de
Salvaguarda são reguladas em seus limites quantitativos ou
temporais.
Acceso comercial. Medida solicitada por qualquer exportador
que se sinta prejudicado por medidas protecionistas de outro pais.
Derechos aduaneros. Barreras arancelarias. Impostos
ou taxas colocados pelo fisco sobre movimentação comercial extra
fronteiras (importação ou exportação)
Desgravación, recorte o rebaja de aranceles. Processo
de redução gradual ou imediata, total ou parcial dos produtos
submetidos a tributação aduaneira.



PROVA DE TRF 2003

El fracaso de Cancún y el futuro de ALCA:

El bloque opositor conformado por Brasil y el grupo de los 20 que
provocó el colapso de las negociaciones de la OMC en la reunión
de Cancún, no es visto por Fernando de Mateo, jefe de la Unidad
de negociaciones para América Latina y el Caribe de la Secretaría
de Economía de México, como una amenaza para la integración
latinoamericana. Aunque Brasil ha mostrado su rechazo al
Acuerdo de Libre Comercio de las Américas (ALCA) y se ha
negado a formar un Tratado de Libre Comercio con México dentro
del MERCOSUR, México y el ALCA están en las prioridades de
Lula.
.
El equipo negociador mexicano se propone reactivar el interés
brasileño, primero para elevar de rango a TLC el Acuerdo de
Complementación que hoy se tiene con ese país sudamericano y,
posteriormente, integrarlo en un mega tratado con el MERCOSUR,
una vez que se concluyan las pláticas con Argentina para un TLC,
sumar el tratado con Uruguay y el tratado que ya se tiene con
Paraguay.
Mateo subrayó que el hecho de que México ya tenga tratados con
la mayoría de los países y que representa la mitad de las
exportaciones totales de América latina, lo coloca en una situación
privilegiada con el ALCA. Dicha región, tiene alrededor de 792
millones de personas, con un PIB de 11 billones de dólares, un
comercio total entre los 34 países del continente de 2,7 billones de
dólares, equivalente a 22% del comercio mundial total. (diario de
Monterrey 22.09.03)

O bloco opositor formado por Brasil e o grupo dos 20 que
provocou o fracasso das negociações da OMC na reunião de
Cancun, não é visto por Mateo, chefe da unidade de negociações
              www.pontodosconcursos.com.br                        3
CURSOS ON-LINE – ESPANHOL – CURSO REGULAR
               PROFESSOR GERARDO SANMARCO
para A. latina e Caribe da secretaria de economia de México,
como uma ameaça para a integração latino americana. Embora
Brasil rechaçou o Acordo de Livre Comércio das Américas (ALCA)
e se negou a formar um TLC com México dentro do Mercosur,
México e a Alca estão nas prioridades de Lula
A equipe negociadora mexicana se propõe reativar o interesse
brasileiro, primeiro para elevar à categoria de TLC o Acordo de
Complementação que hoje tem com esse pais sudamericano e,
posteriormente, integra-lo em um mega tratado com o Mercosur,
quando estejam concluídas as conversações com Argentina para
um TLC, somar o tratado com Uruguai e o tratado que já tem com
Paraguai
Mateo destacou que o fato de que México já tenha tratados com a
maioria dos paises e que representa a metade das exportações
totais de América Latina, o coloca em situação privilegiada com a
Alca. Dita região, tem aproximadamente 792 milhões de pessoas,
com um PIB de 11 trilhões de dólares, um comércio total entre os
34 paises do continente de 2,7 trilhões de dólares, equivalente a
22% do comercio mundial total.

23.- Según Mateo, el fracaso de la reunión de Cancún no
amenaza la integración latinoamericana porque
a) el rechazo de Brasil al ALCA es cuestión pasada
b) Brasil acepta la participación mexicana en el MERCOSUR
c) México y el ALCA están entre las prioridades del gobierno
brasileño
d) México ya tiene tratados con todos los países latinoamericanos
e) el PIB mexicano es el mayor de América

24.- De acuerdo con el texto, la estrategia integracionista
mexicana se caracteriza por
a) incentivos a importaciones oriundas del MERCOSUR
b) el incremento de las exportaciones hacia América latina
c) la remoción de las barreras arancelarias
d) acuerdos individuales en la región y elevación del nivel de los
nexos económicos con Brasil
e) apoyo a las posiciones brasileñas en el seno de la OMC

25.- Según el texto, uno de los aspectos que revelan el peso de la
región del ALCA es el hecho de que:
a) alberga el segundo núcleo consumidor mundial
b) exporta la mitad de toda la producción latinoamericana
c) tiene un PIB de once millones de millones de dólares
d) representa un bloque negociador homogéneo dentro de la OMC
e) incluye a las mayores economías del mundo.

COMENTARIO DO GABARITO DA PROVA 2003

Questão 23
             www.pontodosconcursos.com.br                       4
CURSOS ON-LINE – ESPANHOL – CURSO REGULAR
             PROFESSOR GERARDO SANMARCO
a) Falsa. Brasil continua o rechaço a ALCA
b) Falsa. Brasil não aceita a participação mexicana no
MERCOSUR
c) Verdadeira. Como diz a oração final do primeiro parágrafo,
México y el ALCA estão entre as prioridades do Lula
d) Falsa. México tem tratados con a mayoría, não todos os países
latinoamericanos
e) Falsa. As exportações e não o PIB mexicano é o maior de
América

Questão 24
a) Falsa. A estratégia mexicana não é unilateral
b) Falsa. O incremento das exportações é conseqüência da
estrategia integracionista.
c) Falsa. O enunciado é muito radical. Na verdade, os paises
querem integracionismo mas são muito cautelosos para não
quebrar suas próprias empresas.
d) Verdadeira. O texto menciona textualmente vários acordos
individuais (bilaterais) na região e elevação do nível (rango) dos
nexos econômicos com Brasil
e) Falso. O texto não menciona isso.


Questão 25.- O peso da região ALCA é o fato de que:
a) Falsa. O texto não menciona isso
b) Falsa. É México e não ALCA quem exporta metade de toda a
produção latino americana
c) Verdadeiro. 11 billones = 11.000.000.000.000
                                  millones   de   millones
d) Falso. Não há menção à tal homogeneidade na OMC
e) Falso. América não inclui as maiores economias do mundo.

PROVA DA ESAF 2002/2
La UE y Japón ante el plan agrícola de EE.UU.

La UE y Japón dieron un portazo a la propuesta estadounidense
para reducir del 62 al 25% de media los aranceles a los productos
agrícolas en un plazo de cinco años. El plan presentado por
EEUU, con el apoyo de Canadá y Australia, en la reunión de
ministros de agricultura de las cinco potencias, celebrada en la
ciudad japonesa de Nara, propone una profunda reforma del
sector agrícola, en un período de 5 años.
La propuesta estadounidense supone la utilización de una fórmula
de reducción de aranceles –conocida como la fórmula suiza-, que
recortará más los aranceles más elevados que los bajos y
garantizará que ningún arancel sea superior a un 25% al cabo de
los cinco años. Esas reducciones se efectuarían sobre los
aranceles que realmente se aplican y no sobre los máximos
             www.pontodosconcursos.com.br                       5
CURSOS ON-LINE – ESPANHOL – CURSO REGULAR
              PROFESSOR GERARDO SANMARCO
autorizados, según Washington, que se queja de que, aunque los
aranceles se han reducido en todo el mundo en los últimos años,
los niveles máximos permitidos por la OMC siguen siendo muy
altos (arancel medio de un 62% en la agricultura) mientras que en
EEUU la media es de un 12%.
En cuanto al apoyo, EE UU propone simplificar el actual sistema
de cálculo utilizando una fórmula que limite el uso de ese tipo de
apoyo siempre y cuando sea distorsionante del comercio a un 5%
del valor total de la producción agrícola. También aboga por
eliminar las empresas de comercio estatales porque abusan de su
condición de monopolio y quiere que se supriman los derechos y
privilegios especiales en sus ventas al extranjero y forzarlas a un
régimen de transparencia; así como prohibir los impuestos a la
exportación de productos agrícolas, aunque se exceptuaría de esa
prohibición a los países en desarrollo a no ser que esos
gravámenes no sirvieran para generar ingresos al fisco.
Representantes de Bruselas dejaron entrever que, con su jugada,
Washington trata de distraer la atención sobre las multimillonarias
subvenciones que EEUU ha estado dando a sus agricultores. El
comisario europeo de Agricultura y Pesca extrañó que estas
propuestas “exijan mayor esfuerzo al resto de los países que a EE
UU” a la vez que ese gobierno evita cualquier compromiso para
reducir en su país los apoyos a la exportación, que distorsionan el
mercado o para disminuir el abuso de ayudas internas practicado.
Elpaís 27.07.02

Tradução ao português

A UE e Japão fecharam a porta à proposta norte americana para
reduzir de 62% ao 25% em média, os impostos aduaneiros aos
produtos agrícolas em um prazo de 5 anos. O plano apresentado
por estados Unidos , com o apoio de Canadá e Austrália, na
reunião de ministros de agricultura das cinco potencias, celebrada
na cidade japonesa de Nara, propõe uma profunda reforma do
setor agrícola, em um período de 5 anos.
A proposta norte americana supõe a utilização de uma fórmula de
redução de impostos aduaneiros (chamada de fórmula suíça) que
reduzirá mais os impostos mais altos que os mais baixos e
garantirá que nenhum imposto aduaneiro seja superior a 25% ao
final de 5 anos. Essa reduções se efetuariam sobre os impostos
que realmente se aplicam e não sobre os máximos autorizados,
segundo Washington, que se queixa que, embora os impostos se
reduziram em todo o mundo nos últimos anos, os níveis máximos
autorizados pela OMC continuam muito altos (imposto médio de
62% na agricultura) enquanto que em EEUU a média é de 12%.
Em relação ao apoio, Estados Unidos propõe simplificar o atual
sistema de cálculo utilizando uma fórmula que limite o uso de esse
tipo de apoio (se ele distorcer o comércio) até um 5% do valor total
da produção agrícola. Também defende eliminar as empresas de
               www.pontodosconcursos.com.br                       6
CURSOS ON-LINE – ESPANHOL – CURSO REGULAR
              PROFESSOR GERARDO SANMARCO
comercio estatais porque abusam de sua condição de monopólio e
que se suprimam os direitos e privilégios especiais em suas
vendas ao exterior e força-las a um regime de transparência;
assim como proibir os impostos às exportações de produtos
agrícolas, embora estriam isentos dessa proibição os paises em
desenvolvimento exceto que esse impostos não servissem para
gerar receita do Tesouro.
Representantes de Bruxelas deixaram entrever que com sua
jogada, Estados Unidos tenta distrair a atenção sobre os
multimilionários subsídios que deram a seus agricultores. O
Comissário Europeu de Agricultura e Pesca achou estranho que
estas propostas “exijam maior esforço ao resto dos paises que a
Estados Unidos” ao mesmo tempo que esse governo evita
qualquer compromisso para reduzir em seu próprio pais os apoios
à exportação que distorcem o mercado ou para diminuir o abuso
de ajudas internas praticado.

26. En el texto se dice que el plan norteamericano para la
agricultura:
a) elimina los subsidios a la exportación y reduce los aranceles y
medidas de apoyo interno.
b) reduce los aranceles en un 25%
c) disminuye el apoyo interno a la exportación en un 5%.
d) garantiza el apoyo económico y administrativo estatal a las
exportaciones.
e) favorece el comercio agrícola de los países en desarrollo.

27.- De acuerdo con el texto, la expresión “dieron un portazo”
significa:
a) dieron un espaldarazo
b) echaron una mano
c) rechazaron
d) recibieron con beneplácito
e) aceptaron a regañadientes

28.- Según el texto, la prohibición de impuestos a la exportación
de productos agrícolas:
a) tiene la anuencia europea y japonesa
b) es válida para todos los países miembros de la OMC
c) se orienta hacia las empresas de comercio estatales.
d) no se aplica en países en desarrollo donde esos impuestos
sirvan para generar ingresos del erario.
e) busca equilibrar el comercio agrícola internacional.


COMENTARIO DO GABARITO DA PROVA 2002/2
Questão 26. En el texto se dice que el plan norteamericano para la
agricultura:
a) Verdadeira: é o resumo das três medidas propostas
              www.pontodosconcursos.com.br                       7
CURSOS ON-LINE – ESPANHOL – CURSO REGULAR
              PROFESSOR GERARDO SANMARCO
b) Falso . Propõe a redução a 25% e não em 25%
c) Falso. Propoe diminuir o apoio a um 5% da produção agrícola
total e não em 5%.
d) Falso. Propõe eliminar as tradings estatais que subsidiam aos
exportadores.
e) Falso. Se bem é verdade que uma proposta desse tipo poderia
favorecer aos paises em desenvolvimento, isto não é mencionado
no texto como sendo o objetivo principal da proposta.

Questão 27.- A expressão “dieron un portazo” significa fechar a
porta de forma radical o que em sentido mais direto poderia se
denominar recusa, rechaço (observe-se que o final “azo” de
“portazo” é um aumentativo para indicar a força com que alguma
ação é feita):
a) Falso. O final “azo” de “espaldarazo” rima com “portazo” mas o
resto da palabra significa o contrário “respaldar” significa apoiar e
“dar um espaldarazo” significa dar um grande apoio.
b) Falso. “echaron una mano” significa ajudaram
c) Verdadeira.”Rechazaron” significa que recusaram a proposta
como pode ser comprovado no final do texto de forma clara.
d) Falso. “recibieron con beneplácito” significa que receberam com
agrado
e) Falso “Aceptaron a regañadientes” significa que aceitaram com
má vontade. Como se ve depois, eles não aceitaram a proposta.

Questão 28.- (questão complicada por causa de uma oração
extremadamente longa, com varias negações duplas):
a) Falsa. Nada da proposta americana tem concordância
(anuência) européia e japonesa
b) Falsa. Não vale para todos os países, pois há exceções.
c) Falsa Essa parte da proposta nada tem a ver com as estatais.
d) Verdadeira. A proposta americana visa preservar a receita
tributária da quais alguns paises em desenvolvimento,
marcadamente exportadores, são dependentes.
e) Falsa. Igual que a “e” da questão anterior, a proposta
americana poderia ser deduzida como uma manobra para atrair o
apoio dos paises pobres, mas nada consta explicitamente no
texto.


PROVA DA ESAF 2000

La OMC pide acabar con las ayudas agrícolas

Bruselas. Los primeros borradores de la declaración que debatirá
la Organización del Comercio (OMC) en Seatle (EEUU) propugnan
la supresión de las ayudas agrícolas a la exportación, dijeron


              www.pontodosconcursos.com.br                         8
CURSOS ON-LINE – ESPANHOL – CURSO REGULAR
              PROFESSOR GERARDO SANMARCO
fuentes comunitarias. También prevén reducir “sustancialmente”
los subsidios internos que den lugar a distorsiones en el comercio.
El secretario de la OMC es el encargado de elaborar el proyecto
de declaración, que será debatido por el consejo general los
próximos 4 y 5 de noviembre, antes de ser presentado el 30 de
noviembre en la reunión ministerial en EEUU, con al que dará
comienzo la llamada Ronda del Milenio.
El borrador no recoge aspiraciones de la UE, como son que se
tenga en cuenta en la negociación el carácter “multifuncional” de la
agricultura comunitaria y los problemas derivados del desarrollo de
la biotecnología, que han dado lugar a la polémica sobre alimentos
transgénicos. La OMC quiere que la reducción de las ayudas a la
exportación se acompañe de un compromiso para su eliminación,
así como mayores rebajas en los derechos aduaneros de todos
los productos agrícolas.

Os primeiros rascunhos da declaração que debaterá a OMC em
Seatle defendem a supressão das ajudas agrícolas à exportação,
falaram fontes da união européia. Também prevêem reduzir
substancialmente os subsídios internos que dão lugar às
distorções no comércio
O secretário da OMC é o encarregado de elaborar o prometo de
declaração que será debatido no conselho geral nos próximos 4 e
5 de novembro, antes de ser apresentado o 30 de novembro na
reunião ministerial em Estados Unidos, com a que dará começo a
Rodada do Milênio.
O rascunho não acolhe aspirações da União Européia, como são
que se tenha em consideração na negociação o caráter
multifuncional da agricultura da União Européia e os problemas
derivados do desenvolvimento da biotecnologia, que deram lugar
à polêmica sobre alimentos transgênicos. A OMC quer que a
redução das ajudas à exportação se acompanhe de um
compromisso para sua eliminação, assim como maiores reduções
nos impostos aduaneiros de todos os produtos agrícolas.



20.- En el texto, se dice que los primeros borradores de la
declaración :

a) favorecen la inclusión de ayudas agrícolas a la exportación
b) abogan por la abolición de las ayudas agrícolas a la
   exportación
c) propulsan la supresión de auxilios agrícolas a la exportación
d) propenden a la suspensión de las ayudas agrícolas a la
   exportación
e) cuestionan la anulación de ayudas agrícolas a la exportación

21 En el texto, la palabra “borrador” significa
              www.pontodosconcursos.com.br                         9
CURSOS ON-LINE – ESPANHOL – CURSO REGULAR
           PROFESSOR GERARDO SANMARCO

a)   proyecto
b)   estudio
c)   encuesta
d)   minuta
e)   presupuesto

22.- En el texto se dice que

a) La UE pretende incluir en la negociación la polémica sobre
   alimentos transgénicos
b) La OMC defiende el estímulo al desarrollo biotecnológico
c) La UE aspira a que se aumente la cantidad de productos
   agrícolas que circulen en el mercado común comunitario
d) La OMC rechaza la restricción de subsidios que provoquen
   distorsiones en el comercio
e) La OMC quiere que los productos entren en los países
   pagando tasas menores.


GABARITO COMENTADO DA PROVA 2000

Questão 20
a) Falso. È justamente o oposto.
b) A alternativa tem pouca diferença com a certa que é a “c” . A
pequena diferença está no verbo “propulsam” (de ação) que é
mais adequado que “abogan”(de argumentação) ou “propenden”
(de inclinação).
c) Verdadeira.
d) Falsa. A palavra “propendem” (tem tendência) é muito tênue.
Tampouco se fala de”suspensão” e sim de supressão (eliminação)
e) Falsa. A palavra “cuestionam” é muito fraca para identificar a
posição da OMC que é totalmente definida contra esse tipo de
ajudas.

Questão 21.
a) Falsa. No caso como se trata de uma declaração e não um
projeto, a palavra “borrador” não se ajusta. Se fosse algo anterior
a um projeto seria “anteproyecto”
b) Falsa. A palavra “estudo” não corresponde com rascunho.
c) Falsa. “Encuesta” significa pesquisa.
d) Verdadeira. Trata-se de um texto eliminar que será colocado
em debate
e) Falsa. “Presupuesto” significa orçamento.

Questão 22
a) Falsa. O texto diz que a UE não foi contemplada com a
inclusão desse assunto.

              www.pontodosconcursos.com.br                      10
CURSOS ON-LINE – ESPANHOL – CURSO REGULAR
              PROFESSOR GERARDO SANMARCO
b) Falsa. O texto não cita posição da OMC sobre isso.
c) Falsa. A UE é a principal opositora do livre comercio de
produtos agrícolas.
d) Falsa. É o contrário = A OMC rechaça os subsídios que
provocam distorções no comércio.
e) Verdadeira. A OMC quiere a retirada das restrições
alfandegárias ao comércio internacional.


SIMULADO 29 (Sobre TLC)
                 Colombia firmó el TLC con EEUU

Colombia y Estados Unidos firmaron este miércoles el Tratado de
Libre Comercio (TLC) que contempla la reducción de aranceles y
establece normas comunes para la protección a la propiedad
intelectual y a los inversionistas, entre otras disposiciones. La
firma llega después de 14 rondas de conversaciones durante 22
meses, negociaciones que Colombia empezó junto con Perú y
Ecuador.

"Ésta es una victoria simbólica para la democracia en Colombia",
dijo el "número dos" en materia de comercio exterior de EEUU,
John Veroneau, quien suscribió el acuerdo en nombre de su país.

Por el lado colombiano, la firma fue del ministro de Comercio,
Industria y Turismo, Jorge Humberto Botero, quien también resaltó
el impacto del acuerdo trasciende el ámbito económico.

"Este tratado es de gran importancia en el marco de la lucha
denodada y sin cuartel que libra Colombia, con el decidido apoyo
de Estados Unidos, contra el tráfico de drogas y el terrorismo",
afirmó Botero en el acto, que tuvo lugar en el Banco
Interamericano   de     Desarrollo   (BID),   en    Washington.

Perú ya firmó y ratificó su propio acuerdo, que precisa de la
aprobación del Congreso de EEUU para su entrada en vigor,
mientras que el proceso entre Quito y Washington está en
suspenso. El TLC entre Colombia y EEUU se comenzará a aplicar
una vez que logre el visto bueno de las legislaturas de ambos
países,    así   como      de     la   Corte     Constitucional.

Tras la ceremonia, Botero dijo esperar que el Congreso de EEUU
pueda iniciar el debate sobre el TLC "lo antes posible". Para
Colombia, la principal ventaja del acuerdo es un acceso "seguro"
al mayor mercado del mundo, según destacó Veroneau.

Actualmente, la mayoría de las exportaciones de Colombia entra

             www.pontodosconcursos.com.br                     11
CURSOS ON-LINE – ESPANHOL – CURSO REGULAR
            PROFESSOR GERARDO SANMARCO
en EEUU sin pagar aranceles gracias a la llamada Ley de
Preferencias Arancelarias Andinas y Erradicación de Drogas
(ATPDEA). Pero este privilegio es temporal y depende de la
renovación por el Congreso, como se pone de manifiesto
actualmente, dado que la Ley expira en diciembre próximo.

En cambio, para Estados Unidos el TLC eliminará las barreras a
la participación de empresas de su país en el sector de los
servicios en Colombia, protegerá la propiedad intelectual de sus
farmacéuticas y otras compañías, y establecerá un mecanismo
para la resolución de disputas de inversión, dijo Veroneau.
Además, facilitará la entrada de las exportaciones de EEUU en un
mercado de 44 millones de personas.1/12/2006

1.- En el texto se dice que el TLC
a) establece acuerdos comerciales entre EEUU, Colombia,
Ecuador y Perú.
b) recorta derechos aduaneros y cobija derechos de inversores
c) fue culminado tras 14 interrupciones de negociaciones
d) dicta normas eliminando impuestos al comercio exterior
e) ha sido confirmado por los Congresos de los países signatarios

2.- Segundo Botero, el acuerdo
a) simboliza los avances de la democracia colombiana
b) permitirá la victoria sobre el terrorismo y el tráfico de drogas
c) refuerza la posición de Colombia en el Banco Interamericano de
Desarrollo
d) va más allá de lo meramente económico
e) dará a Colombia un acceso al mayor país del mundo

3.- En el texto se dice que
a) El congreso americano debatirá el TLC cuanto antes
b) el TLC no modifica en nada los acuerdos comerciales ya
existentes
c) el TLC será temporal y sujeto a permanentes renovaciones del
Congreso
d) los productos colombianos nunca pagaron arancel para entrar
en EEUU
e) el TLC tiene varias etapas antes de entrar en vigor

4.- En el textos se dice que en el TLC
a) se establecieron acuerdos iguales para las dos partes
b) hay facilidades para las ventas externas de EEUU a Colombia
c) se establece el acceso irrestricto de las empresas
estadounidenses en Colombia.
d) se establece un mercado de 44 millones como límite de las
ventajas
e) favorecerá más a EEUU de que a Colombia

             www.pontodosconcursos.com.br                       12
CURSOS ON-LINE – ESPANHOL – CURSO REGULAR
             PROFESSOR GERARDO SANMARCO
5.- La expresión “que logre el visto bueno” equivale en ese
contexto a
a) que siga con una buena visión
b) que se debata poco rato
c) que obtenga dictamen favorable
d) que consiga ser soslayado
e) que mitigue las contradiciones

6.- La expresión “en cambio” equivale en ese contexto a
a) Tras la transformación
b) Cuando cambie
c) Dentro de la alteración
d) Por el contrario
e) En suma

7.- La expresión “además” equivale en ese contexto a
a) asimismo
b) por demás
c) por ello
d) en cuanto a
e) sin embargo

FICHA GRAMATICAL 14
SOBRE NEXOS INTERPARÁGRAFOS
Eles são peças fundamentais para dar coesão a um texto.
Eles estabelecem conexões de diversos tipos conforme o autor
queira vincular as idéias e os argumentos enumerados.
A seguir, veremos os principais , com sinônimos e traduções

Asimismo = además = también. Indica que os parágrafos estão
associados pela fonte ou pelo conteúdo.
De hecho = en efecto = efectivamente. Indica que o parágrafo
seguinte é confirmação do anterior, detalhado através de
fatos ou exemplos práticos.

Por ello = por lo tanto = de ahí que. Indica que o parágrafo
seguinte é conseqüência do anterior.

En cambio = por el contrario =por otra parte. Indica que o
parágrafo seguinte é muito diferente do anterior no
conteúdo informativo

En suma = en resumen. Indica que o parágrafo seguinte é um
resumo dos conceitos e/ou argumentos mencionados nos
parágrafos anteriores.

             www.pontodosconcursos.com.br                 13
CURSOS ON-LINE – ESPANHOL – CURSO REGULAR
          PROFESSOR GERARDO SANMARCO

Sin embargo = empero = no obstante. Indica que o parágrafo
seguinte desenvolverá informações e/ou argumentações
opostas às do parágrafo anterior.




SIMULADO 30 (sobre MERCOSUR)

Inmersos en un debate sobre los retos del mayor mecanismo de
integración del área, los cancilleres de Argentina, Brasil, Uruguay
y Paraguay, más Venezuela, celebran en Buenos Aires la V
Reunión Extraordinaria del Consejo del Mercado Común (CMC). El
encuentro del CMC, principal instancia de decisión del
MERCOSUR, avanza en la incorporación de Caracas como socio
pleno del bloque sureño, el tema que concentrará la atención de
los mandatarios los días 20 y 21 de julio venideros. La cita,
llamada por Argentina en su condición de presidente temporal de
la unión aduanera, función que traspasará a Brasil en el segundo
semestre, debate, además, una posición común con miras a la
próxima reunión de la Organización Mundial de Comercio. Los
ministros también evalúan las asimetrías que afectan el comercio
dentro del MERCOSUR, en particular a Uruguay y Paraguay, las
economías más débiles de este mecanismo de concertación.
Según lo previsto, en el cónclave de mandatarios se suscribirán
los protocolos de adhesión de la República Bolivariana al bloque
sureño como su quinto miembro oficial. El ingreso de Venezuela,
 considerada la tercera economía de Sudamérica, es vista al
interior de la asociación como un factor de fortalecimiento del
proceso de integración real en el Cono Sur, pese a todas sus
limitaciones y debilidades.
Luego de su firma, esos documentos deberán ser sancionados por
los parlamentos de Venezuela y de los cuatro socios fundadores
de la unión aduanera. El borrador del pacto fija un plazo máximo
de cuatro años a partir de su ratificación para que Caracas adopte
el arancel externo común y el acervo normativo del MERCOSUR.
Establece, además, plazos que llegan hasta 2013 para el libre
comercio entre la República Bolivariana y el resto de los miembros
de la unión aduanera sudamericana. Fuentes diplomáticas
argentinas explicaron que en los protocolos se fija un cronograma
de desgravación arancelaria por parte de la nación andina, el cual
será de inmediata aplicación para Uruguay y Paraguay, los socios
menores del bloque. Para el caso de Argentina y Brasil, las
exenciones tendrán un carácter gradual y con plazos variables
hasta el 2018, con lo cual se logró un reconocimiento explícito de
las asimetrías comerciales dentro del MERCOSUR.
             www.pontodosconcursos.com.br                       14
CURSOS ON-LINE – ESPANHOL – CURSO REGULAR
          PROFESSOR GERARDO SANMARCO

1.- Según el texto, la reunión del Consejo del MERCOSUR en julio
a) concluye el proceso de integración de Venezuela como quinto
miembro
b) decide quien asumirá la presidencia pre-tempore del
MERCOSUR
c) analiza los desequilibrios comerciales en el seno del bloque
d) observa la situación de los países débiles que comercian con el
MERCOSUR
e) incluye pláticas para la incorporación a la OMC

2.- En el texto se dice en relación a la integración de Venezuela
a) tendrá hasta 2010 para adecuarse al acervo de resoluciones del
MERCOSUR
b) tendrá que ser apoyada en los 5 suscritores por sendas
mayorías parlamentares
c) tendrá que implantar el arancel externo a humo de pajas.
d) que serán pesadas sus debilidades y fuerzas para integrar el
bloque
e) que es la tercera mayor economía del continente americano

3.-En relación al borrador que será evaluado en la reunión del CM,
él plantea que Venezuela
a) adhiera al imposto común de importación externa al
MERCOSUR en cuatro años
b) implante de forma completa su arancel externo común con
Uruguay y Paraguay hasta 2013
c) implante su arancel externo común con Brasil y Argentina en 12
años
d) Venezuela conceda exenciones aduaneras irrestrictas a sus
socios del MERCOSUR
e) tiene cuatro años para iniciar la desgravación de aranceles con
sus socios del MERCOSUR.

4.-El término “desgravación arancelaria” (li.23) se refiere
específicamente a
a) los acuerdos para reglamentación de la base imponible
b) decisiones presupuestarias
c) retiradas de las barreras impositivas y sanitarias a la
importación
d) rebaja gradual o total de tributos al comercio exterior
e) exoneración de gravámenes a la actividad económica en los
países miembros

5.- La expresión “pese a” (li.18) en ese contexto equivale a
a) por causa de
b) sin embargo de
c) sino por
d) mismo con
              www.pontodosconcursos.com.br                     15
CURSOS ON-LINE – ESPANHOL – CURSO REGULAR
             PROFESSOR GERARDO SANMARCO
e) siempre y cuando

6. La expresión “luego de” (li.19) en el texto equivale a
a) apués
b) en seguida
c) conque
d) a menudo
e) tras

7.-La expresión “los retos” (li.10) en el texto equivale a
a) los acuerdos
b) los hechos
c) los desafíos
d) los desencuentros
e) tropiezos

8.- La expresión “con miras a” (li.9) significa
a) precaviéndose de
b) de ojo en
c) soslayando
d) sintonizándose con
e) interfiriendo en

SIMULADO 31 (sobre China)

China es un socio “muy importante” para el desarrollo de América
Latina, dijo hoy en Hong Kong el diplomático chileno Miguel
Poklepovic. El chileno explicó que “el hecho es que China es muy
importante para el desarrollo de América Latina, mientras que al
contrario, eso no es tan obvio”. En la actualidad, China,
fervientemente interesada en las materias primas que ofrece la
región, ha encontrado en el nuevo continente un socio de primer
nivel. Y no sólo eso, sino que la balanza comercial entre ambas
partes ascendió a 400.000 millones de dólares en el año 2004, y
fue especialmente importante en los casos de México, Brasil y
Panamá.
“No sólo el comercio entre China y América Latina seguirá
aumentando, sino que casi que el 50% de las inversiones de
China van a ir a parar allí”, dijo a Efe Poklepovic. Según fuentes
oficiales, el 49.3% de las inversiones chinas a nivel mundial del
año 2004 estuvieron destinadas a América Latina, por encima
incluso de las que las empresas chinas dedicaron a Hong Kong.
“Es cierto que ha habido un cambio en los intereses de China,
pero eso se hubiera quedado en agua de borrajas si no hubiera
sido recíproco por parte de América Latina”, aseguró.
 “Ahora (los chinos) poseen no sólo derechos de suministro, sino
también minas, en Chile o en Perú”, dijo el diplomático. Desde que
China alcanzó sus más altos niveles de desarrollo, en las últimas
décadas, ha dirigido hacia América Latina, así como hacia el resto
               www.pontodosconcursos.com.br                     16
CURSOS ON-LINE – ESPANHOL – CURSO REGULAR
              PROFESSOR GERARDO SANMARCO
del Tercer Mundo, una “diplomacia de los recursos”. No obstante,
según los expertos, el creciente interés de los asiáticos por ese
continente no representa una amenaza para Washington, quien ha
dominado tradicionalmente la zona.
Sin embargo, Poklepovic aseguró que el petróleo no es uno de
esos recursos estratégicos, ya que las exportaciones del oro negro
de América Latina a China se circunscriben a pequeños acuerdos
con Venezuela, Cuba y Brasil. Sí lo son, por el contrario, alimentos
como la soja procedentes de Brasil y de Argentina. En la
actualidad, “Argentina y Brasil proporcionan el 29% de las
importaciones totales de comida de China”.
Además, China tiene también importantes intereses políticos en la
zona. En la actualidad doce de los 26 países que todavía
reconocen al gobierno de Taipei son latinoamericanos, lo que es
un “factor irritante” para China a la hora de tratar con bloques
comerciales como Mercosur, en el que se incluye Paraguay.

1.- En el texto se dice que
a) en 2004, el intercambio comercial China/América Latina
aumentó en 400 mil millones de dólares
b) casi el 50% del comercio exterior chino está dirigido a América
Latina
c) América Latina es reacia al intercambio comercial con China
d) los chinos controlan los recursos minerales de Chile y otros
países
e) el desplazamiento de capitales chinos al exterior tiene como
meta principal A. Latina

2.- En el texto se dice que
a) hay un entredicho diplomático entre China y Paraguay por
causa de Taipei
b) el MERCOSUR irrita a China por motivos económicos
c) Argentina y Brasil son responsables por 30% del consumo
alimentar de China
d) el interés de China por A. Latina apunta a sus necesidades
energéticas
e) China realiza pequeñas importaciones de oro procedente de A.
Latina

3.- En ese contexto, la expresión “derechos de suministro” (li.14)
equivale a
a) tasas de importación
b) reservas de compra
c) depósitos portuarios
d) empresas comercializadoras
e) estoques de reserva

4.- En ese contexto, la expresión “agua de borrajas” tiene el
sentido de
            www.pontodosconcursos.com.br                   17
CURSOS ON-LINE – ESPANHOL – CURSO REGULAR
              PROFESSOR GERARDO SANMARCO
a) tempestuoso
b) sospechoso
c) insostenible
d) sin importancia
e) inexplicable


Lecturas Atualizadas sobre
OMC, blocos e TLCs
Texto 1 (Sobre el TLC Colombia EEUU)

La situación textil y de la vestimenta podría agravarse aún más en
los próximos meses, y tanto desde el sector industrial como desde
el gobierno se buscan soluciones urgentes. Para el presidente de
la Cámara Industrial de la Vestimenta, Elbio Fuscaldo, se está en
un momento "bisagra" (dobradiça) y de transición, en un mercado
de signado por una continua suba de los costos y las pérdidas de
los mercados externos. Los factores principales que enfrenta la
industria uruguaya en el contexto externo son la desaparición de la
cuotificación que China tenía para exportar a Estados Unidos y
ahora todas las empresas ponen su mirada en ese gigante
asiático. La exportación ha caído a unos U$S 60 millones por año.
A esto se suma un hecho (um fato) no menor: el acuerdo de Libre
Comercio entre Estados Unidos y Colombia, que se ha convertido
en la nueva cuna(berço) de la vestimenta en Latinoamérica, no
sólo con una mano de obra barata, sino con tecnología, moda y
diseño. Actualmente Colombia y Estados Unidos tienen un
sistema de preferencias que será sustituido por el TLC. Pero ese
sistema ya llevó a que la industria de la vestimenta uruguaya se
alejara (afastasse) del principal consumidor del mundo. Mientras
que Uruguay debe pagar un arancel de ingreso de 18%, Colombia
no paga ni pagará aranceles. Es por esta razón que el ministro de
Industria, Jorge Lepra, señaló en varias ocasiones que el sector
textil sería el más afectado si no se firmaba un TLC con Estados
Unidos, porque Colombia está a punto de convertirse en su
principal competidor de ropa de calidad y a bajo precio. Porque si
bien la industria textil exporta, gran parte de ella debe de vivir del
mercado interno, el cual se ve saturado por ropa que ingresa
desde Argentina y en especial desde China. - La subfacturación
en el ingreso de mercadería y el contrabando también están
afectando al sector.(larepublica.uy 25.11.2006)

Texto 2

Bogotá. - El Parlamento Andino se declaró hoy preocupado por el
Tratado de Libre Comercio (TLC) firmado por Colombia y Perú con
Estados Unidos al advertir que en las discusiones para concretar
              www.pontodosconcursos.com.br                         18
CURSOS ON-LINE – ESPANHOL – CURSO REGULAR
               PROFESSOR GERARDO SANMARCO
la alianza hubo "aspectos que pasaron desapercibidos" y por ello
solicitó a los países andinos realizar un debate más profundo de la
negociación.

El presidente del Parlamento Andino, el senador colombiano Luís
Fernando Duque, insistió en la necesidad de analizar
minuciosamente los acuerdos realizados con el gobierno
estadounidense para evitar que los tratados generen un
desequilibrio en "el bienestar de nuestros pueblos y el crecimiento
de nuestras economías", citó DPA.

"Es indispensable que los congresos de Colombia y Perú
escuchen a los representantes del órgano deliberante de la
Comunidad Andina (CAN). Hemos adelantando una serie de
análisis sobre los efectos del TLC en la normativa comercial y la
institucionalidad del proceso andino de integración y tenemos
mucho que aportar al debate al interior de los congresos sobre los
efectos del TLC en la CAN", expresó Duque.

Asimismo, recordó que en el texto actual del TLC negociado entre
Bogotá y Washington no hay claridad sobre el tema agrícola que
quedó sin salvaguardias, franjas de precios o productos excluidos
"sin ser muy claro el acceso real al mercado de Estados Unidos en
condiciones de competitividad".

"El proceso andino de integración va mucho más allá de los
aspectos comerciales, como no sucede con el TLC con Estados
Unidos; la CAN se ha preocupado por promover y atender los
aspectos sociales y culturales de los ciudadanos andinos, por
esto, se debe analizar este Tratado Comercial a la luz de lo que se
ha construido en 37 años de integración andina", agregó.

Duque se declaró partidario de los acuerdos comerciales con
terceros pero insistió en la necesidad de que estas alianzas
comerciales "fortalezcan la capacidad competitiva de nuestros
aparatos productivos y ayuden a mejorar el nivel de vida de los
pueblos andinos".

Los gobiernos de Estados Unidos y Colombia firmaron ayer en la
sede del Banco Interamericano de Desarrollo en Washington un
Tratado de Libre Comercio y quedaron a la espera de su
aprobación en los respectivos congresos, donde se espera
afrontará (enfrentará) los más duros cuestionamientos.

Por su parte, el tratado con Perú, impulsado por el ex presidente
Alejandro Toledo y apoyado por el del actual mandatario, Alan
García, ya fue firmado por los ejecutivos de Lima y Washington y


             www.pontodosconcursos.com.br                       19
CURSOS ON-LINE – ESPANHOL – CURSO REGULAR
            PROFESSOR GERARDO SANMARCO
refrendado por el Congreso peruano, pero falta que el Legislativo
estadounidense lo apruebe.

Texto 3

Negociaciones sobre subsídios bloqueadas

GINEBRA.- El Grupo de los Veinte (G-20), que reúne a países en
desarrollo, anunció hoy que responderá a las iniciativas de la UE y
EEUU para desbloquear las negociaciones agrícolas de la
Organización Mundial de Comercio (OMC) con una "propuesta
propia y con números concretos" que se referirá de forma concreta
a los tres pilares de la negociación agrícola, como son los
subsidios a la exportación, el apoyo interno y el acceso a los
mercados", dijo el ministro brasileño Celso Amorim.
"En apoyo interno (a los agricultores) buscamos medidas que
supongan un auténtico recorte de esos niveles, y en acceso a
mercados, verdaderas oportunidades para acceder a ellos, con
recortes como los que hubo en la Ronda de Uruguay", precisó
Amorim.
Agregó que para el G-20 es esencial que "en 2010 se eliminen
todas las formas de apoyo a las exportaciones y las que se dan a
nivel nacional" en la agricultura.
Las diferencias en agricultura, uno de los capítulos claves de esta
negociación, bloquean el resto de los acuerdos sobre servicios,
acceso a mercados para productos industrializados o facilitación
comercial, entre otros.
El Representante Especial de Comercio de EEUU, Rob Portman,
presentó este lunes una iniciativa que afecta a los tres pilares
agrícolas y que se ejecutaría en dos etapas, una inicial en la que
aplicarían recortes significativos en los aranceles y en las ayudas
directas en un periodo de cinco años. Cinco años después se
pasaría a otra fase, en la que se procedería a la total eliminación
de las políticas que distorsionan el comercio agrícola.
La UE, por su parte, propuso reducir un 70% las ayudas agrícolas
internas que distorsionan el comercio, las que en el argot de la
OMC se consideran de la "caja azul" o de la "caja amarilla", es
decir que distorsionan el comercio y que están ligadas a la
producción o a los precios. Asimismo, está dispuesta a rebajar el
65% los llamados apoyos que dentro de la OMC están exentos de
rebajas siempre y cuando no superen el 5% del valor total de la
producción agrícola.

Texto 4

GINEBRA.- Las potencias comerciales del mundo hicieron
avances menudos (pequenos)en las últimas décadas en las
iniciativas para reducir los subsidios agrícolas que distorsionan el
comercio, de acuerdo con un estudio publicado el martes por la
               www.pontodosconcursos.com.br                      20
CURSOS ON-LINE – ESPANHOL – CURSO REGULAR
              PROFESSOR GERARDO SANMARCO
OCDE. El informe de la OCDE -que agrupa a las naciones más
desarrolladas y tiene su sede en París- mostró también que
Islandia, Suiza y Noruega mantenían los niveles más altos de
apoyo a los agricultores entre los 30 miembros de la OCDE.
Además, Estados Unidos incrementó sus niveles de protección
en el 2004.
"Las reformas de las políticas se han centrado en cambiar la forma
en la que se ofrece la ayuda a los productores, con un alejamiento
(afastamento)notable de las medidas relacionadas con el volumen
de producción", dijo el reporte en referencia a los pagos que
reciben los granjeros y que están atados a su producción.
No obstante, aunque estas reformas podrían continuar en los
próximos años, dijo la OCDE, la participación de esas formas de
respaldo que tanto distorsionan la producción y el comercio
igualmente representó el 74% de todos los subsidios en el período
2002- 2004. Esto reflejó una reducción desde el 91% del lapso
1986- 1988.
El apoyo agrícola de la UE surgió como un tema importante el fin
de semana pasado cuando fracasó un encuentro de los líderes del
bloque para ponerse de acuerdo sobre el presupuesto de la Unión
Europeia para el período 2007-2013.
La falta de entendimiento en gran medida se debió a que Francia
rechazó el pedido de Gran Bretaña para que se recorten los
subsidios al agro.Esta cuestión también es uno de los ejes(eixos)
de la Ronda de Doha de negociaciones para la liberalización
comercial, un proceso en el seno de la Organización Mundial del
Comercio (OMC) que ha avanzado despacio(devagar). (emol-
22/6/05)

Texto 5 (Conferencia OMC- 2005)

Los países ricos han roto(quebrado), durante la preparación a la
Cumbre de la OMC que se celebra la semana que viene en Hong
Kong, las promesas que hicieron hace cuatro años. El informe de
la ONG Intermón Oxfam muestra cómo al pedir más y más a los
países pobres, a cambio de muy poco, le han dado la espalda (as
costas) a lo que tenía que haber sido la 'Ronda de Desarrollo'.
"Las negociaciones, que deberían haber llevado a una reforma de
las reglas comerciales para potenciar el desarrollo, han
descarrilado(derrapado) por culpa de los tiras y aflojas de los más
ricos afirma Gonzalo Fanjul, coordinador de Intermón Oxfam. "Los
países más desarrollados exigen un gran número de concesiones,
porque dicen que necesitan ver 'sangre en el suelo' de todos para
poder vender el acuerdo en la OMC en sus propios países. Esta
posición de poder le ha dado la vuelta a las promesas hechas
(feitas) en Doha. Con la tendencia actual, las negociaciones tienen
poco que ofrecer a los que más lo necesitan", añade Fanjul.
Fanjul denuncia que la reforma en la agricultura es esencial para
millones de personas, pero los países más ricos no han dado las
              www.pontodosconcursos.com.br                       21
CURSOS ON-LINE – ESPANHOL – CURSO REGULAR
              PROFESSOR GERARDO SANMARCO
pasos para acabar con los subsidios a la exportación ni para
mejorar el acceso a sus mercados. Mientras tanto, los Estados
más pobres reciben presiones para liberalizar sus mercados.
Las negociaciones sobre los aranceles industriales son, si cabe,
más preocupantes. El borrador acordado en julio de 2004 puede
destruir los sectores industriales de muchos países pobres.
Además, les están obligando a recortar sus aranceles por encima
de los que los países ricos pretenden realizar, lo que representa
una contradicción de las promesas hechas en Doha.
Respecto a los servicios, en lugar de respetar el método de
inclusión acordado en Doha, los países ricos están pidiendo la
aplicación de un compromiso mínimo respecto al número de
sectores aceptados y el nivel de apertura, que podría llevar a una
liberalización forzada de sectores sociales básicos de los países
más pobres.
Si el nivel de intransigencia se mantiene, las negociaciones
pueden quedar colapsadas a lo largo de la próxima década. Por
desgracia, las alternativas a un acuerdo fuera de la OMC tampoco
son atractivas. Las negociaciones comerciales regionales o
bilaterales ponen en gran riesgo las promesas de desarrollo.

Texto 6 (Sobre TLC Perú-EEUU)

Perú y Estados Unidos cerraron ayer con un acuerdo sus
negociaciones para un Tratado de Libre Comercio, el primero en
dos años con un país suramericano después de Chile.. El acuerdo
fue logrado bilateralmente, pese a que Perú inició el proceso de
negociación en bloque con Ecuador y Colombia.
Se desconoce de momento si con ello se rompería definitivamente
el frente andino que conformaron esos tres países. Fuentes de la
delegación peruana dijeron que no formularían ningún comentario
sobre el acuerdo antes de que fuera anunciado en Lima por el
presidente Alejandro Toledo.
El presidente George W. Bush deberá notificar al Congreso con 90
días de anticipación su “intención de firmar” el tratado con Perú,
un procedimiento que es rutinario. Pero el envío al Congreso para
ratificación puede complicarse debido a que el próximo año en
noviembre habrá elecciones legislativas y pocos legisladores
desean entrar en un tema que ha tenido tenaces discrepantes.
Estados Unidos ha firmado ya acuerdos de libre comercio con
México, Chile y cinco países centroamericanos más República
Dominicana agrupados en el convenio llamado CAFTA.. Está
adicionalmente negociando con Panamá.
Colombia y Ecuador se retiraron de las negociaciones el 22 de
noviembre en Washington luego de 19 meses de tratos iniciados
en la ciudad colombiana de Cartagena. Los dos países dijeron que


             www.pontodosconcursos.com.br                      22
CURSOS ON-LINE – ESPANHOL – CURSO REGULAR
               PROFESSOR GERARDO SANMARCO
no podían seguir hablando mientras Estados Unidos se mostraba
intransigente en sus peticiones.
Perú, en cambio, pidió continuar las negociaciones en la primera
semana de diciembre, lo cual hizo a partir del lunes en que una
delegación de varios ministros llegó a Washington para negociar
prácticamente 24 horas al día. En su intento de avanzar, el
presidente Alejandro Toledo instruyó a sus negociadores a que
terminen el proceso a como diera lugar, dijeron fuentes
informadas.
Los ministros de Agricultura, Comercio Exterior, Producción y
Salud, que integran el frente negociador peruano, se habían
estado resistiendo a algunas propuestas estadounidenses, ante lo
cual Toledo envió a su hombre de confianza, su primer ministro y
presidente del consejo de ministros Pedro Pablo Kuczynski, para
facilitar un entendimiento. 5/12/2005

Texto 7 (Sobre conflitos de comércio exterior)

BRUSELAS.- La Unión Europea ha vuelto a fracasar en su intento
de buscar un consenso sobre los aranceles al calzado de cuero
importado desde China y Vietnam. Los 25 tendrán una última
oportunidad el próximo miércoles, dos días antes de expire el
actual régimen de tarifas temporales.
Los representantes de los Estados miembros trataron de alcanzar
un acuerdo en base a la última propuesta de la Comisión, que
establece un arancel durante tres años de 16,5% para el calzado
de China y del 10% para el importado de Vietnam.
Éste es el último planteamiento de Bruselas para sustituir al
sistema temporal de aranceles progresivos que empezó a
aplicarse el pasado mes de abril y que en la actualidad alcanza el
19,4% para los zapatos de China y el 16,8% para los de Vietnam.
Este mecanismo, que concluirá el 6 de octubre, dará lugar a la
total liberalización de las importaciones si no se halla un acuerdo
alternativo.
Estos aranceles progresivos y temporales hacen referencia a
medidas 'antidumping' que se impusieron después de que una
investigación de la Comisión Europea concluyera que existe una
fuerte intervención del Estado en los dos países asiáticos que
provoca que los zapatos se vendan en Europa por debajo de su
coste de producción.
Los Estados miembros llevan negociando desde hace meses qué
medidas de 'antidumping' podrían aplicarse a partir del 7 de
octubre, pues los 25 se debaten entre dos posiciones: una, la de
los países productores que, como España o Italia, apuestan por la
imposición de medidas y ven con buenos ojos la última propuesta
de la Comisión, y dos, la de los países con un potente sector
distribuidor, particularmente los nórdicos, que apuestan por la
liberalización.

               www.pontodosconcursos.com.br                     23
CURSOS ON-LINE – ESPANHOL – CURSO REGULAR
               PROFESSOR GERARDO SANMARCO
No obstante, fuentes españolas explicaron que Chipre se ha
sumado al grupo de países productores junto a España, Francia,
Italia, Portugal, Polonia, Lituania y Hungría.
La decisión se tomará por mayoría simple, de manera que, con la
suma de Chipre, solamente haría falta un Estado miembro que
votara con el grupo del que forma parte España o que se
abstuviera para llegar al acuerdo. A este respecto, aventuraron
fuentes españolas que "quizás" Austria o Letonia podrían cambiar
su postura.
Francia, además, formuló una nueva propuesta en aras del nuevo
compromiso, que consiste en que el nuevo régimen de aranceles
se aplique durante dos años en lugar de tres como proponía la
Comisión.
El portavoz de Comercio del Ejecutivo comunitario, Peter Power,
mantuvo la cautela sobre esta nueva propuesta, que primero
tendrá que ser estudiada por el comisario del ramo, Peter
Mandelson, y posteriormente ser aprobada por los 25 comisarios.
"Es un periodo un poco corto pero si los Estados miembros
quieren, entonces tendremos que estar con ellos".


GABARITO SIMULADO 30

1.- En el texto se dice que el TLC
b) recorta derechos aduaneros             (aranceles)     y   cobija
(protege)derechos de inversores

2.- Segundo Botero, el acuerdo
d) va más allá de lo meramente económico

3.- En el texto se dice que
e) el TLC tiene varias etapas antes de entrar en vigor

4.- En el textos se dice que en el TLC
b) hay facilidades para las ventas externas de EEUU a Colombia

5.- La expresión “que logre el visto bueno” equivale en ese
contexto a
c) que obtenga dictamen favorable

6.- La expresión “en cambio” equivale en ese contexto a
d) Por el contrario

7.- La expresión “además” equivale en ese contexto a
a) asimismo


GABARITO SIMULADO 30

              www.pontodosconcursos.com.br                       24
CURSOS ON-LINE – ESPANHOL – CURSO REGULAR
              PROFESSOR GERARDO SANMARCO
1.- Según el texto, la reunión del Consejo del MERCOSUR en julio
c) analiza los desequilibrios comerciales(ASIMETRÍAS)en el seno
del bloque

2.- En el texto se dice en relación a la integración de Venezuela
b) tendrá que ser apoyada en los 5 suscritores (V4ENEZUELA,
BRASIL, Argentina, Paraguay, Uruguay) por sendas mayorías
parlamentares.

3.-En relación al borrador que será evaluado en la reunión del CM,
él plantea que Venezuela
a) adhiera al imposto común de importación externa al
MERCOSUR en menos de cuatro años

4.-El término “desgravación arancelaria” (li.23) se refiere
específicamente a   d) rebaja gradual o total de tributos al
comercio exterior

5.- La expresión “pese a” (li.18) en ese contexto equivale a
d) mismo con

6. La expresión “luego de” (li.19) en el texto equivale a
e) tras

7.-La expresión “los retos” (li.10) en el texto equivale a
c) los desafíos

8.- La expresión “con miras a” (li.9) significa
b) de ojo en

GABARITO SIMULADO 31

1.- En el texto se dice que
e) el desplazamiento(deslocamento) de capitales chinos al exterior
tiene como meta principal A. Latina

2.- En el texto se dice que
a) hay un entredicho diplomático entre China y Paraguay por
causa de Taipei

3.- En ese contexto, la expresión “derechos de suministro” (li.14)
equivale a
b) reservas de compra

4.- En ese contexto, la expresión “agua de borrajas” tiene el
sentido de
d) sin importancia


               www.pontodosconcursos.com.br                    25

Más contenido relacionado

La actualidad más candente

La actualidad más candente (20)

27-02-14 El Futuro del TLCAN tras 20 años de su entrada en vigor
27-02-14 El Futuro del TLCAN tras 20 años de su entrada en vigor27-02-14 El Futuro del TLCAN tras 20 años de su entrada en vigor
27-02-14 El Futuro del TLCAN tras 20 años de su entrada en vigor
 
TRATADO DE LIBRE COMERCIO TLCAN
TRATADO DE LIBRE COMERCIO TLCANTRATADO DE LIBRE COMERCIO TLCAN
TRATADO DE LIBRE COMERCIO TLCAN
 
G3 méxico colombia
G3 méxico colombiaG3 méxico colombia
G3 méxico colombia
 
TLC MEXICO-G3
TLC MEXICO-G3TLC MEXICO-G3
TLC MEXICO-G3
 
Alca (2)
Alca (2)Alca (2)
Alca (2)
 
Ventajas y beneficios de invertir en Estados Unidos
Ventajas y beneficios de invertir en Estados UnidosVentajas y beneficios de invertir en Estados Unidos
Ventajas y beneficios de invertir en Estados Unidos
 
Causas y consecuencias del TLCAN
Causas y consecuencias del TLCANCausas y consecuencias del TLCAN
Causas y consecuencias del TLCAN
 
Alca
AlcaAlca
Alca
 
NAFTA
NAFTANAFTA
NAFTA
 
Bloque nafta y efta
Bloque nafta y eftaBloque nafta y efta
Bloque nafta y efta
 
Presentación mcca(mercado común centro americano)
Presentación mcca(mercado común centro americano)Presentación mcca(mercado común centro americano)
Presentación mcca(mercado común centro americano)
 
MERCOSUR
MERCOSURMERCOSUR
MERCOSUR
 
Diapositivas TLC
Diapositivas TLCDiapositivas TLC
Diapositivas TLC
 
Nafta
NaftaNafta
Nafta
 
G 3 / G2 COLOMBIA Y MEXICO
G 3 / G2 COLOMBIA Y MEXICOG 3 / G2 COLOMBIA Y MEXICO
G 3 / G2 COLOMBIA Y MEXICO
 
Exposicion tlcan. mayra nolasco
Exposicion tlcan. mayra nolascoExposicion tlcan. mayra nolasco
Exposicion tlcan. mayra nolasco
 
Sesion 6 MCCA
Sesion 6 MCCASesion 6 MCCA
Sesion 6 MCCA
 
Tlcan final
Tlcan finalTlcan final
Tlcan final
 
Tlcan2
Tlcan2Tlcan2
Tlcan2
 
Tlcan
TlcanTlcan
Tlcan
 

Destacado

41931065 taller practico n1 s2
41931065 taller practico n1 s241931065 taller practico n1 s2
41931065 taller practico n1 s2astrydquintero
 
Opal belt buckle
Opal belt buckleOpal belt buckle
Opal belt buckleopalmine73
 
If Your Love Was True
If Your Love Was TrueIf Your Love Was True
If Your Love Was TrueNikhil Parekh
 
Invest_in_Childcare__Invest_in_London_-_4in10_report
Invest_in_Childcare__Invest_in_London_-_4in10_reportInvest_in_Childcare__Invest_in_London_-_4in10_report
Invest_in_Childcare__Invest_in_London_-_4in10_reportHannah Slater
 
Asnalis de una experiencia pedagogica aplicada
Asnalis de una experiencia pedagogica aplicadaAsnalis de una experiencia pedagogica aplicada
Asnalis de una experiencia pedagogica aplicadamigrupo1
 
El Programa BIEE: Semana de la Energía Sostenible 2016
El Programa BIEE: Semana de la Energía Sostenible 2016El Programa BIEE: Semana de la Energía Sostenible 2016
El Programa BIEE: Semana de la Energía Sostenible 2016Andres Schuschny, Ph.D
 
Escala conceitos aplic
Escala conceitos aplicEscala conceitos aplic
Escala conceitos aplicRaniere Costa
 
السيرة الذاتية شريف الصاوي
السيرة الذاتية شريف الصاويالسيرة الذاتية شريف الصاوي
السيرة الذاتية شريف الصاويSherif Al-Sawy
 
Nerd News | Media Kit 2017
Nerd News | Media Kit 2017Nerd News | Media Kit 2017
Nerd News | Media Kit 2017Marcelo Nunes
 
Apresentacao sobre comunicacao no iiam
Apresentacao sobre comunicacao no iiamApresentacao sobre comunicacao no iiam
Apresentacao sobre comunicacao no iiamSostino Mocumbe
 
Efecto del uso del frio y embalaje en la deshidratación de frutos
Efecto del uso del frio y embalaje en la deshidratación de frutosEfecto del uso del frio y embalaje en la deshidratación de frutos
Efecto del uso del frio y embalaje en la deshidratación de frutosJhonás A. Vega
 

Destacado (20)

41931065 taller practico n1 s2
41931065 taller practico n1 s241931065 taller practico n1 s2
41931065 taller practico n1 s2
 
GREETS-MASTER PROFILE
GREETS-MASTER PROFILEGREETS-MASTER PROFILE
GREETS-MASTER PROFILE
 
Nelly Osorio Godoy
Nelly Osorio Godoy	Nelly Osorio Godoy
Nelly Osorio Godoy
 
Opal belt buckle
Opal belt buckleOpal belt buckle
Opal belt buckle
 
Clase sobre Microficción
Clase sobre MicroficciónClase sobre Microficción
Clase sobre Microficción
 
If Your Love Was True
If Your Love Was TrueIf Your Love Was True
If Your Love Was True
 
Invest_in_Childcare__Invest_in_London_-_4in10_report
Invest_in_Childcare__Invest_in_London_-_4in10_reportInvest_in_Childcare__Invest_in_London_-_4in10_report
Invest_in_Childcare__Invest_in_London_-_4in10_report
 
Prueba
PruebaPrueba
Prueba
 
Asnalis de una experiencia pedagogica aplicada
Asnalis de una experiencia pedagogica aplicadaAsnalis de una experiencia pedagogica aplicada
Asnalis de una experiencia pedagogica aplicada
 
El Programa BIEE: Semana de la Energía Sostenible 2016
El Programa BIEE: Semana de la Energía Sostenible 2016El Programa BIEE: Semana de la Energía Sostenible 2016
El Programa BIEE: Semana de la Energía Sostenible 2016
 
Engine heat recovery
Engine heat recoveryEngine heat recovery
Engine heat recovery
 
Escala conceitos aplic
Escala conceitos aplicEscala conceitos aplic
Escala conceitos aplic
 
السيرة الذاتية شريف الصاوي
السيرة الذاتية شريف الصاويالسيرة الذاتية شريف الصاوي
السيرة الذاتية شريف الصاوي
 
Nerd News | Media Kit 2017
Nerd News | Media Kit 2017Nerd News | Media Kit 2017
Nerd News | Media Kit 2017
 
Blog
BlogBlog
Blog
 
Rosas
RosasRosas
Rosas
 
Palestra hq
Palestra hqPalestra hq
Palestra hq
 
Manual radiomobile
Manual radiomobileManual radiomobile
Manual radiomobile
 
Apresentacao sobre comunicacao no iiam
Apresentacao sobre comunicacao no iiamApresentacao sobre comunicacao no iiam
Apresentacao sobre comunicacao no iiam
 
Efecto del uso del frio y embalaje en la deshidratación de frutos
Efecto del uso del frio y embalaje en la deshidratación de frutosEfecto del uso del frio y embalaje en la deshidratación de frutos
Efecto del uso del frio y embalaje en la deshidratación de frutos
 

Similar a Espanhol regular 11

Diapositivas (alca)
Diapositivas (alca)Diapositivas (alca)
Diapositivas (alca)Tuxneoii
 
Tratado de libre comercio
Tratado de libre comercioTratado de libre comercio
Tratado de libre comercioDAOSs
 
IMPACTO DEL TLC EN COLOMBIA
IMPACTO DEL TLC EN COLOMBIAIMPACTO DEL TLC EN COLOMBIA
IMPACTO DEL TLC EN COLOMBIADumar Castillo
 
2000_05_Apertura al exterior y negociaciones comerciales
2000_05_Apertura al exterior y negociaciones comerciales2000_05_Apertura al exterior y negociaciones comerciales
2000_05_Apertura al exterior y negociaciones comercialesCarlos Mladinic
 
UNSADA Comex - Módulo 1.pptx
UNSADA Comex - Módulo 1.pptxUNSADA Comex - Módulo 1.pptx
UNSADA Comex - Módulo 1.pptxmarcos406401
 
El Tratado de Libre Comercio (TLCAN) de México con Estados Unidos y Canadá a ...
El Tratado de Libre Comercio (TLCAN) de México con Estados Unidos y Canadá a ...El Tratado de Libre Comercio (TLCAN) de México con Estados Unidos y Canadá a ...
El Tratado de Libre Comercio (TLCAN) de México con Estados Unidos y Canadá a ...Economist
 
Ensayo 2 acuerdos_comerciales[1] paula
Ensayo 2 acuerdos_comerciales[1] paulaEnsayo 2 acuerdos_comerciales[1] paula
Ensayo 2 acuerdos_comerciales[1] paulapaulash726
 
Negociaciones en Materia de Política Comercial
Negociaciones en Materia de Política ComercialNegociaciones en Materia de Política Comercial
Negociaciones en Materia de Política ComercialProyecto AdA-Integración
 
Ec. internacional clase 15 integración economica
Ec. internacional   clase 15 integración economicaEc. internacional   clase 15 integración economica
Ec. internacional clase 15 integración economicaCarlos Rojas
 

Similar a Espanhol regular 11 (20)

Diapositivas (alca) 1
Diapositivas (alca) 1Diapositivas (alca) 1
Diapositivas (alca) 1
 
plan ALCA
plan ALCAplan ALCA
plan ALCA
 
Diapositivas (alca)
Diapositivas (alca)Diapositivas (alca)
Diapositivas (alca)
 
Diapositivas (alca)
Diapositivas (alca)Diapositivas (alca)
Diapositivas (alca)
 
Los desafíos del Comercio Internacional
Los desafíos del Comercio InternacionalLos desafíos del Comercio Internacional
Los desafíos del Comercio Internacional
 
Tratado de libre comercio
Tratado de libre comercioTratado de libre comercio
Tratado de libre comercio
 
omc
omcomc
omc
 
Acuerdos comerciales
Acuerdos comercialesAcuerdos comerciales
Acuerdos comerciales
 
IMPACTO DEL TLC EN COLOMBIA
IMPACTO DEL TLC EN COLOMBIAIMPACTO DEL TLC EN COLOMBIA
IMPACTO DEL TLC EN COLOMBIA
 
2000_05_Apertura al exterior y negociaciones comerciales
2000_05_Apertura al exterior y negociaciones comerciales2000_05_Apertura al exterior y negociaciones comerciales
2000_05_Apertura al exterior y negociaciones comerciales
 
Guía de ejercicios chile en el mundo actual
Guía de ejercicios chile en el mundo actualGuía de ejercicios chile en el mundo actual
Guía de ejercicios chile en el mundo actual
 
UNSADA Comex - Módulo 1.pptx
UNSADA Comex - Módulo 1.pptxUNSADA Comex - Módulo 1.pptx
UNSADA Comex - Módulo 1.pptx
 
El Tratado de Libre Comercio (TLCAN) de México con Estados Unidos y Canadá a ...
El Tratado de Libre Comercio (TLCAN) de México con Estados Unidos y Canadá a ...El Tratado de Libre Comercio (TLCAN) de México con Estados Unidos y Canadá a ...
El Tratado de Libre Comercio (TLCAN) de México con Estados Unidos y Canadá a ...
 
Exposicion02 politica economica_globalizacion
Exposicion02 politica economica_globalizacionExposicion02 politica economica_globalizacion
Exposicion02 politica economica_globalizacion
 
Exposicion02 politica economica_globalizacion
Exposicion02 politica economica_globalizacionExposicion02 politica economica_globalizacion
Exposicion02 politica economica_globalizacion
 
Exposicion02 politica economica_globalizacion
Exposicion02 politica economica_globalizacionExposicion02 politica economica_globalizacion
Exposicion02 politica economica_globalizacion
 
Ensayo 2 acuerdos_comerciales[1] paula
Ensayo 2 acuerdos_comerciales[1] paulaEnsayo 2 acuerdos_comerciales[1] paula
Ensayo 2 acuerdos_comerciales[1] paula
 
Negociaciones en Materia de Política Comercial
Negociaciones en Materia de Política ComercialNegociaciones en Materia de Política Comercial
Negociaciones en Materia de Política Comercial
 
MI_01SUBIR.pdf
MI_01SUBIR.pdfMI_01SUBIR.pdf
MI_01SUBIR.pdf
 
Ec. internacional clase 15 integración economica
Ec. internacional   clase 15 integración economicaEc. internacional   clase 15 integración economica
Ec. internacional clase 15 integración economica
 

Más de J M

Português 2008
Português   2008Português   2008
Português 2008J M
 
Português 2011
Português   2011Português   2011
Português 2011J M
 
Classes das palavras
Classes das palavrasClasses das palavras
Classes das palavrasJ M
 
Unprotected apostila-matematica
Unprotected apostila-matematicaUnprotected apostila-matematica
Unprotected apostila-matematicaJ M
 
Unprotected apostila português
Unprotected apostila portuguêsUnprotected apostila português
Unprotected apostila portuguêsJ M
 
Unprotected apostila conhecimentos gerais
Unprotected apostila conhecimentos geraisUnprotected apostila conhecimentos gerais
Unprotected apostila conhecimentos geraisJ M
 
Unprotected apostila conhecimentos bancários
Unprotected apostila conhecimentos bancáriosUnprotected apostila conhecimentos bancários
Unprotected apostila conhecimentos bancáriosJ M
 
Tecnicas de redacao
Tecnicas de redacaoTecnicas de redacao
Tecnicas de redacaoJ M
 
Raciocinio lógico exercícios resolvidos
Raciocinio lógico   exercícios resolvidosRaciocinio lógico   exercícios resolvidos
Raciocinio lógico exercícios resolvidosJ M
 
Raciocínio lógico aula 6-6 - diagramas lógicos
Raciocínio lógico   aula 6-6 - diagramas lógicosRaciocínio lógico   aula 6-6 - diagramas lógicos
Raciocínio lógico aula 6-6 - diagramas lógicosJ M
 
Raciocínio lógico aula 5-6 - estruturas lógicas 2
Raciocínio lógico   aula 5-6 - estruturas lógicas 2Raciocínio lógico   aula 5-6 - estruturas lógicas 2
Raciocínio lógico aula 5-6 - estruturas lógicas 2J M
 
Raciocínio lógico aula 3-6 - lógica de argumentaçao
Raciocínio lógico   aula 3-6 - lógica de argumentaçaoRaciocínio lógico   aula 3-6 - lógica de argumentaçao
Raciocínio lógico aula 3-6 - lógica de argumentaçaoJ M
 
Raciocínio lógico aula 1-6 - conceitos iniciais 1
Raciocínio lógico   aula 1-6 - conceitos iniciais 1Raciocínio lógico   aula 1-6 - conceitos iniciais 1
Raciocínio lógico aula 1-6 - conceitos iniciais 1J M
 
Raciocínio lógico aula 0-6 - orientaçoes iniciais - questoes sem gabarito
Raciocínio lógico   aula 0-6 - orientaçoes iniciais - questoes sem gabaritoRaciocínio lógico   aula 0-6 - orientaçoes iniciais - questoes sem gabarito
Raciocínio lógico aula 0-6 - orientaçoes iniciais - questoes sem gabaritoJ M
 
Prova senado (raciocínio lógico)
Prova senado (raciocínio lógico)Prova senado (raciocínio lógico)
Prova senado (raciocínio lógico)J M
 
Ministério público (12 anos de provas em concurso)
Ministério público (12 anos de provas em concurso)Ministério público (12 anos de provas em concurso)
Ministério público (12 anos de provas em concurso)J M
 
Matemática para concursos provas gabaritadas
Matemática para concursos   provas gabaritadasMatemática para concursos   provas gabaritadas
Matemática para concursos provas gabaritadasJ M
 
Manual do desempregado
Manual do desempregadoManual do desempregado
Manual do desempregadoJ M
 
Estatistica concursos esaf
Estatistica concursos esafEstatistica concursos esaf
Estatistica concursos esafJ M
 
Direito constitucional provas receita federal - 130 ques
Direito constitucional   provas receita federal - 130 quesDireito constitucional   provas receita federal - 130 ques
Direito constitucional provas receita federal - 130 quesJ M
 

Más de J M (20)

Português 2008
Português   2008Português   2008
Português 2008
 
Português 2011
Português   2011Português   2011
Português 2011
 
Classes das palavras
Classes das palavrasClasses das palavras
Classes das palavras
 
Unprotected apostila-matematica
Unprotected apostila-matematicaUnprotected apostila-matematica
Unprotected apostila-matematica
 
Unprotected apostila português
Unprotected apostila portuguêsUnprotected apostila português
Unprotected apostila português
 
Unprotected apostila conhecimentos gerais
Unprotected apostila conhecimentos geraisUnprotected apostila conhecimentos gerais
Unprotected apostila conhecimentos gerais
 
Unprotected apostila conhecimentos bancários
Unprotected apostila conhecimentos bancáriosUnprotected apostila conhecimentos bancários
Unprotected apostila conhecimentos bancários
 
Tecnicas de redacao
Tecnicas de redacaoTecnicas de redacao
Tecnicas de redacao
 
Raciocinio lógico exercícios resolvidos
Raciocinio lógico   exercícios resolvidosRaciocinio lógico   exercícios resolvidos
Raciocinio lógico exercícios resolvidos
 
Raciocínio lógico aula 6-6 - diagramas lógicos
Raciocínio lógico   aula 6-6 - diagramas lógicosRaciocínio lógico   aula 6-6 - diagramas lógicos
Raciocínio lógico aula 6-6 - diagramas lógicos
 
Raciocínio lógico aula 5-6 - estruturas lógicas 2
Raciocínio lógico   aula 5-6 - estruturas lógicas 2Raciocínio lógico   aula 5-6 - estruturas lógicas 2
Raciocínio lógico aula 5-6 - estruturas lógicas 2
 
Raciocínio lógico aula 3-6 - lógica de argumentaçao
Raciocínio lógico   aula 3-6 - lógica de argumentaçaoRaciocínio lógico   aula 3-6 - lógica de argumentaçao
Raciocínio lógico aula 3-6 - lógica de argumentaçao
 
Raciocínio lógico aula 1-6 - conceitos iniciais 1
Raciocínio lógico   aula 1-6 - conceitos iniciais 1Raciocínio lógico   aula 1-6 - conceitos iniciais 1
Raciocínio lógico aula 1-6 - conceitos iniciais 1
 
Raciocínio lógico aula 0-6 - orientaçoes iniciais - questoes sem gabarito
Raciocínio lógico   aula 0-6 - orientaçoes iniciais - questoes sem gabaritoRaciocínio lógico   aula 0-6 - orientaçoes iniciais - questoes sem gabarito
Raciocínio lógico aula 0-6 - orientaçoes iniciais - questoes sem gabarito
 
Prova senado (raciocínio lógico)
Prova senado (raciocínio lógico)Prova senado (raciocínio lógico)
Prova senado (raciocínio lógico)
 
Ministério público (12 anos de provas em concurso)
Ministério público (12 anos de provas em concurso)Ministério público (12 anos de provas em concurso)
Ministério público (12 anos de provas em concurso)
 
Matemática para concursos provas gabaritadas
Matemática para concursos   provas gabaritadasMatemática para concursos   provas gabaritadas
Matemática para concursos provas gabaritadas
 
Manual do desempregado
Manual do desempregadoManual do desempregado
Manual do desempregado
 
Estatistica concursos esaf
Estatistica concursos esafEstatistica concursos esaf
Estatistica concursos esaf
 
Direito constitucional provas receita federal - 130 ques
Direito constitucional   provas receita federal - 130 quesDireito constitucional   provas receita federal - 130 ques
Direito constitucional provas receita federal - 130 ques
 

Espanhol regular 11

  • 1. CURSOS ON-LINE – ESPANHOL – CURSO REGULAR PROFESSOR GERARDO SANMARCO Aula 11 OMC, Blocos regionais e TLCs Hola amigos!. O cenário do comercio internacional se apresenta extremamente dinâmico. Se, por um lado, os acordos mundiais de liberalização de comércio via OMC estão estagnados diante da posição protecionista da União Européia, os blocos regionais continuam tentando processos de maior integração (Can e Mercosur). Em 2006, o processo que vem alcançando maior repercussão é o da multiplicação de TLCs, tratados bilaterais de liberalização comercial. Em nosso continente, os Estado Unidos (que parecem ter desistido provisoriamente da ALCA) o México e o Chile despontam como os paises que tem aproveitado com mais intensidade esses processos para estimular o crescimento das suas economias. Do ponto de vista da Fazenda Pública, os processos de liberalização comercial despertam particular preocupação, porque eles vêm baseados em redução dos impostos de importação, e portanto em perda de receita tributária. Considerando que o parecer técnico da perda de receita tributária nunca será argumento suficiente para contrabalancear os benefícios de uma possível melhora do crescimento econômico nacional, o Ministério de Fazenda deve repensar as estratégias para preparar mini reformas tributárias que salvaguardem as necessidades estratégicas do Tesouro. A ESAF vem acompanhando esse cenário mundial, e tem colocado textos vinculados a ele, em quase todas as provas de 2000 em diante. O concursando precisa acompanhar muito de perto as notícias de atualidade, e dominar o vocabulário conceitual a ele relacionado para não ser tomado por surpresa no próximo concurso. Recomendamos especialmente a leitura permanente de noticias em espanhol que marcam a evolução de • As negociações da OMC (particularmente do G20) • A evolução do Nafta • A evolução do CAFTA • A evolução do Mercosur • A evolução da CAN • A evolução da União Européia • A evolução da Zona Euro • Os TLC em negociação ou em vigência próxima. Nesta aula retomaremos alguns textos de provas recentes da ESAF, simulados e leituras que confluem para reforçar os conhecimentos nessa área. Antes de iniciar os estudos www.pontodosconcursos.com.br 1
  • 2. CURSOS ON-LINE – ESPANHOL – CURSO REGULAR PROFESSOR GERARDO SANMARCO recomendamos revisar alguns conceitos que aparecerão nas provas, simulados e leituras que completam esta aula: FICHA DE VOCABULARIO 10 MAPA CONCEITUAL SOBRE COMERCIO MUNDIAL E TRIBUTAÇÃO ADUANEIRA OMC = Organização Mundial do Comércio reúne quase 150 paises foi criada para regular o comércio mundial. Tem alguns órgãos como o Tribunal de Conflitos que emite pareceres sobre disputas comerciais. Tem como filosofia a redução gradativa dos obstáculos (tributários e sanitários) que prejudicam o comercio mundial e agravam a pobreza das nações menos desenvolvidas. Convoca conferencias mundiais cada poucos anos para tentar arrancar acordos dos blocos mais protecionistas: Ronda do Milênio (Seatle 2000), Ronda de Doha (Emirados 2001), Conferencia de Cancun (2003), Conferencia de Hong Kong (2005). Os grandes temas que enfrentam os eixos Norte/Sur são: 1.- Desgravación arancelaria. Os paises do G20 exigem a diminuição dos aranceles (impostos aduaneiros) que pesam sobre Os produtos agrícolas exportáveis. Os paises do G8 exigem como contrapartida que os paises do G20 reduzam o protecionismo sobre os produtos industrializados e a quebra das patentes da industrias farmacêuticas. 2.-Ayudas agrícolas . Apoyo interno.-Os paises do G20 exigem o fim dos subsídios (dinheiro a custo zero) que recebem os agricultores europeus para sobreviver à concorrência das exportações dos paises do G20. 3.- Subsidios a exportaciones. O G20 e a União Européia exigem que Estados Unidos, Japão e China, acabem com as Tradings (exportadoras) estatais que praticam subsídios mascarados aos exportadores para compensar sua falta de competitividade no mercado externo. Outros termos: Subvenciones. Apoio em dinheiro que agricultores e/ou exportadores recém do Estado como compensação diante da concorrência com outros paises. A OMC o considera “dumping” ou concorrência desleal” e permite aos paises vítimas, o direito de retaliação. Salvaguardas o Derechos compensatórios. Medidas de retaliação ( impostos adicionais ou quotas alfandegárias) permitidas pela OMC quando um pais é vitima de www.pontodosconcursos.com.br 2
  • 3. CURSOS ON-LINE – ESPANHOL – CURSO REGULAR PROFESSOR GERARDO SANMARCO “dumping” por parte das exportações de outro. As medidas de Salvaguarda são reguladas em seus limites quantitativos ou temporais. Acceso comercial. Medida solicitada por qualquer exportador que se sinta prejudicado por medidas protecionistas de outro pais. Derechos aduaneros. Barreras arancelarias. Impostos ou taxas colocados pelo fisco sobre movimentação comercial extra fronteiras (importação ou exportação) Desgravación, recorte o rebaja de aranceles. Processo de redução gradual ou imediata, total ou parcial dos produtos submetidos a tributação aduaneira. PROVA DE TRF 2003 El fracaso de Cancún y el futuro de ALCA: El bloque opositor conformado por Brasil y el grupo de los 20 que provocó el colapso de las negociaciones de la OMC en la reunión de Cancún, no es visto por Fernando de Mateo, jefe de la Unidad de negociaciones para América Latina y el Caribe de la Secretaría de Economía de México, como una amenaza para la integración latinoamericana. Aunque Brasil ha mostrado su rechazo al Acuerdo de Libre Comercio de las Américas (ALCA) y se ha negado a formar un Tratado de Libre Comercio con México dentro del MERCOSUR, México y el ALCA están en las prioridades de Lula. . El equipo negociador mexicano se propone reactivar el interés brasileño, primero para elevar de rango a TLC el Acuerdo de Complementación que hoy se tiene con ese país sudamericano y, posteriormente, integrarlo en un mega tratado con el MERCOSUR, una vez que se concluyan las pláticas con Argentina para un TLC, sumar el tratado con Uruguay y el tratado que ya se tiene con Paraguay. Mateo subrayó que el hecho de que México ya tenga tratados con la mayoría de los países y que representa la mitad de las exportaciones totales de América latina, lo coloca en una situación privilegiada con el ALCA. Dicha región, tiene alrededor de 792 millones de personas, con un PIB de 11 billones de dólares, un comercio total entre los 34 países del continente de 2,7 billones de dólares, equivalente a 22% del comercio mundial total. (diario de Monterrey 22.09.03) O bloco opositor formado por Brasil e o grupo dos 20 que provocou o fracasso das negociações da OMC na reunião de Cancun, não é visto por Mateo, chefe da unidade de negociações www.pontodosconcursos.com.br 3
  • 4. CURSOS ON-LINE – ESPANHOL – CURSO REGULAR PROFESSOR GERARDO SANMARCO para A. latina e Caribe da secretaria de economia de México, como uma ameaça para a integração latino americana. Embora Brasil rechaçou o Acordo de Livre Comércio das Américas (ALCA) e se negou a formar um TLC com México dentro do Mercosur, México e a Alca estão nas prioridades de Lula A equipe negociadora mexicana se propõe reativar o interesse brasileiro, primeiro para elevar à categoria de TLC o Acordo de Complementação que hoje tem com esse pais sudamericano e, posteriormente, integra-lo em um mega tratado com o Mercosur, quando estejam concluídas as conversações com Argentina para um TLC, somar o tratado com Uruguai e o tratado que já tem com Paraguai Mateo destacou que o fato de que México já tenha tratados com a maioria dos paises e que representa a metade das exportações totais de América Latina, o coloca em situação privilegiada com a Alca. Dita região, tem aproximadamente 792 milhões de pessoas, com um PIB de 11 trilhões de dólares, um comércio total entre os 34 paises do continente de 2,7 trilhões de dólares, equivalente a 22% do comercio mundial total. 23.- Según Mateo, el fracaso de la reunión de Cancún no amenaza la integración latinoamericana porque a) el rechazo de Brasil al ALCA es cuestión pasada b) Brasil acepta la participación mexicana en el MERCOSUR c) México y el ALCA están entre las prioridades del gobierno brasileño d) México ya tiene tratados con todos los países latinoamericanos e) el PIB mexicano es el mayor de América 24.- De acuerdo con el texto, la estrategia integracionista mexicana se caracteriza por a) incentivos a importaciones oriundas del MERCOSUR b) el incremento de las exportaciones hacia América latina c) la remoción de las barreras arancelarias d) acuerdos individuales en la región y elevación del nivel de los nexos económicos con Brasil e) apoyo a las posiciones brasileñas en el seno de la OMC 25.- Según el texto, uno de los aspectos que revelan el peso de la región del ALCA es el hecho de que: a) alberga el segundo núcleo consumidor mundial b) exporta la mitad de toda la producción latinoamericana c) tiene un PIB de once millones de millones de dólares d) representa un bloque negociador homogéneo dentro de la OMC e) incluye a las mayores economías del mundo. COMENTARIO DO GABARITO DA PROVA 2003 Questão 23 www.pontodosconcursos.com.br 4
  • 5. CURSOS ON-LINE – ESPANHOL – CURSO REGULAR PROFESSOR GERARDO SANMARCO a) Falsa. Brasil continua o rechaço a ALCA b) Falsa. Brasil não aceita a participação mexicana no MERCOSUR c) Verdadeira. Como diz a oração final do primeiro parágrafo, México y el ALCA estão entre as prioridades do Lula d) Falsa. México tem tratados con a mayoría, não todos os países latinoamericanos e) Falsa. As exportações e não o PIB mexicano é o maior de América Questão 24 a) Falsa. A estratégia mexicana não é unilateral b) Falsa. O incremento das exportações é conseqüência da estrategia integracionista. c) Falsa. O enunciado é muito radical. Na verdade, os paises querem integracionismo mas são muito cautelosos para não quebrar suas próprias empresas. d) Verdadeira. O texto menciona textualmente vários acordos individuais (bilaterais) na região e elevação do nível (rango) dos nexos econômicos com Brasil e) Falso. O texto não menciona isso. Questão 25.- O peso da região ALCA é o fato de que: a) Falsa. O texto não menciona isso b) Falsa. É México e não ALCA quem exporta metade de toda a produção latino americana c) Verdadeiro. 11 billones = 11.000.000.000.000 millones de millones d) Falso. Não há menção à tal homogeneidade na OMC e) Falso. América não inclui as maiores economias do mundo. PROVA DA ESAF 2002/2 La UE y Japón ante el plan agrícola de EE.UU. La UE y Japón dieron un portazo a la propuesta estadounidense para reducir del 62 al 25% de media los aranceles a los productos agrícolas en un plazo de cinco años. El plan presentado por EEUU, con el apoyo de Canadá y Australia, en la reunión de ministros de agricultura de las cinco potencias, celebrada en la ciudad japonesa de Nara, propone una profunda reforma del sector agrícola, en un período de 5 años. La propuesta estadounidense supone la utilización de una fórmula de reducción de aranceles –conocida como la fórmula suiza-, que recortará más los aranceles más elevados que los bajos y garantizará que ningún arancel sea superior a un 25% al cabo de los cinco años. Esas reducciones se efectuarían sobre los aranceles que realmente se aplican y no sobre los máximos www.pontodosconcursos.com.br 5
  • 6. CURSOS ON-LINE – ESPANHOL – CURSO REGULAR PROFESSOR GERARDO SANMARCO autorizados, según Washington, que se queja de que, aunque los aranceles se han reducido en todo el mundo en los últimos años, los niveles máximos permitidos por la OMC siguen siendo muy altos (arancel medio de un 62% en la agricultura) mientras que en EEUU la media es de un 12%. En cuanto al apoyo, EE UU propone simplificar el actual sistema de cálculo utilizando una fórmula que limite el uso de ese tipo de apoyo siempre y cuando sea distorsionante del comercio a un 5% del valor total de la producción agrícola. También aboga por eliminar las empresas de comercio estatales porque abusan de su condición de monopolio y quiere que se supriman los derechos y privilegios especiales en sus ventas al extranjero y forzarlas a un régimen de transparencia; así como prohibir los impuestos a la exportación de productos agrícolas, aunque se exceptuaría de esa prohibición a los países en desarrollo a no ser que esos gravámenes no sirvieran para generar ingresos al fisco. Representantes de Bruselas dejaron entrever que, con su jugada, Washington trata de distraer la atención sobre las multimillonarias subvenciones que EEUU ha estado dando a sus agricultores. El comisario europeo de Agricultura y Pesca extrañó que estas propuestas “exijan mayor esfuerzo al resto de los países que a EE UU” a la vez que ese gobierno evita cualquier compromiso para reducir en su país los apoyos a la exportación, que distorsionan el mercado o para disminuir el abuso de ayudas internas practicado. Elpaís 27.07.02 Tradução ao português A UE e Japão fecharam a porta à proposta norte americana para reduzir de 62% ao 25% em média, os impostos aduaneiros aos produtos agrícolas em um prazo de 5 anos. O plano apresentado por estados Unidos , com o apoio de Canadá e Austrália, na reunião de ministros de agricultura das cinco potencias, celebrada na cidade japonesa de Nara, propõe uma profunda reforma do setor agrícola, em um período de 5 anos. A proposta norte americana supõe a utilização de uma fórmula de redução de impostos aduaneiros (chamada de fórmula suíça) que reduzirá mais os impostos mais altos que os mais baixos e garantirá que nenhum imposto aduaneiro seja superior a 25% ao final de 5 anos. Essa reduções se efetuariam sobre os impostos que realmente se aplicam e não sobre os máximos autorizados, segundo Washington, que se queixa que, embora os impostos se reduziram em todo o mundo nos últimos anos, os níveis máximos autorizados pela OMC continuam muito altos (imposto médio de 62% na agricultura) enquanto que em EEUU a média é de 12%. Em relação ao apoio, Estados Unidos propõe simplificar o atual sistema de cálculo utilizando uma fórmula que limite o uso de esse tipo de apoio (se ele distorcer o comércio) até um 5% do valor total da produção agrícola. Também defende eliminar as empresas de www.pontodosconcursos.com.br 6
  • 7. CURSOS ON-LINE – ESPANHOL – CURSO REGULAR PROFESSOR GERARDO SANMARCO comercio estatais porque abusam de sua condição de monopólio e que se suprimam os direitos e privilégios especiais em suas vendas ao exterior e força-las a um regime de transparência; assim como proibir os impostos às exportações de produtos agrícolas, embora estriam isentos dessa proibição os paises em desenvolvimento exceto que esse impostos não servissem para gerar receita do Tesouro. Representantes de Bruxelas deixaram entrever que com sua jogada, Estados Unidos tenta distrair a atenção sobre os multimilionários subsídios que deram a seus agricultores. O Comissário Europeu de Agricultura e Pesca achou estranho que estas propostas “exijam maior esforço ao resto dos paises que a Estados Unidos” ao mesmo tempo que esse governo evita qualquer compromisso para reduzir em seu próprio pais os apoios à exportação que distorcem o mercado ou para diminuir o abuso de ajudas internas praticado. 26. En el texto se dice que el plan norteamericano para la agricultura: a) elimina los subsidios a la exportación y reduce los aranceles y medidas de apoyo interno. b) reduce los aranceles en un 25% c) disminuye el apoyo interno a la exportación en un 5%. d) garantiza el apoyo económico y administrativo estatal a las exportaciones. e) favorece el comercio agrícola de los países en desarrollo. 27.- De acuerdo con el texto, la expresión “dieron un portazo” significa: a) dieron un espaldarazo b) echaron una mano c) rechazaron d) recibieron con beneplácito e) aceptaron a regañadientes 28.- Según el texto, la prohibición de impuestos a la exportación de productos agrícolas: a) tiene la anuencia europea y japonesa b) es válida para todos los países miembros de la OMC c) se orienta hacia las empresas de comercio estatales. d) no se aplica en países en desarrollo donde esos impuestos sirvan para generar ingresos del erario. e) busca equilibrar el comercio agrícola internacional. COMENTARIO DO GABARITO DA PROVA 2002/2 Questão 26. En el texto se dice que el plan norteamericano para la agricultura: a) Verdadeira: é o resumo das três medidas propostas www.pontodosconcursos.com.br 7
  • 8. CURSOS ON-LINE – ESPANHOL – CURSO REGULAR PROFESSOR GERARDO SANMARCO b) Falso . Propõe a redução a 25% e não em 25% c) Falso. Propoe diminuir o apoio a um 5% da produção agrícola total e não em 5%. d) Falso. Propõe eliminar as tradings estatais que subsidiam aos exportadores. e) Falso. Se bem é verdade que uma proposta desse tipo poderia favorecer aos paises em desenvolvimento, isto não é mencionado no texto como sendo o objetivo principal da proposta. Questão 27.- A expressão “dieron un portazo” significa fechar a porta de forma radical o que em sentido mais direto poderia se denominar recusa, rechaço (observe-se que o final “azo” de “portazo” é um aumentativo para indicar a força com que alguma ação é feita): a) Falso. O final “azo” de “espaldarazo” rima com “portazo” mas o resto da palabra significa o contrário “respaldar” significa apoiar e “dar um espaldarazo” significa dar um grande apoio. b) Falso. “echaron una mano” significa ajudaram c) Verdadeira.”Rechazaron” significa que recusaram a proposta como pode ser comprovado no final do texto de forma clara. d) Falso. “recibieron con beneplácito” significa que receberam com agrado e) Falso “Aceptaron a regañadientes” significa que aceitaram com má vontade. Como se ve depois, eles não aceitaram a proposta. Questão 28.- (questão complicada por causa de uma oração extremadamente longa, com varias negações duplas): a) Falsa. Nada da proposta americana tem concordância (anuência) européia e japonesa b) Falsa. Não vale para todos os países, pois há exceções. c) Falsa Essa parte da proposta nada tem a ver com as estatais. d) Verdadeira. A proposta americana visa preservar a receita tributária da quais alguns paises em desenvolvimento, marcadamente exportadores, são dependentes. e) Falsa. Igual que a “e” da questão anterior, a proposta americana poderia ser deduzida como uma manobra para atrair o apoio dos paises pobres, mas nada consta explicitamente no texto. PROVA DA ESAF 2000 La OMC pide acabar con las ayudas agrícolas Bruselas. Los primeros borradores de la declaración que debatirá la Organización del Comercio (OMC) en Seatle (EEUU) propugnan la supresión de las ayudas agrícolas a la exportación, dijeron www.pontodosconcursos.com.br 8
  • 9. CURSOS ON-LINE – ESPANHOL – CURSO REGULAR PROFESSOR GERARDO SANMARCO fuentes comunitarias. También prevén reducir “sustancialmente” los subsidios internos que den lugar a distorsiones en el comercio. El secretario de la OMC es el encargado de elaborar el proyecto de declaración, que será debatido por el consejo general los próximos 4 y 5 de noviembre, antes de ser presentado el 30 de noviembre en la reunión ministerial en EEUU, con al que dará comienzo la llamada Ronda del Milenio. El borrador no recoge aspiraciones de la UE, como son que se tenga en cuenta en la negociación el carácter “multifuncional” de la agricultura comunitaria y los problemas derivados del desarrollo de la biotecnología, que han dado lugar a la polémica sobre alimentos transgénicos. La OMC quiere que la reducción de las ayudas a la exportación se acompañe de un compromiso para su eliminación, así como mayores rebajas en los derechos aduaneros de todos los productos agrícolas. Os primeiros rascunhos da declaração que debaterá a OMC em Seatle defendem a supressão das ajudas agrícolas à exportação, falaram fontes da união européia. Também prevêem reduzir substancialmente os subsídios internos que dão lugar às distorções no comércio O secretário da OMC é o encarregado de elaborar o prometo de declaração que será debatido no conselho geral nos próximos 4 e 5 de novembro, antes de ser apresentado o 30 de novembro na reunião ministerial em Estados Unidos, com a que dará começo a Rodada do Milênio. O rascunho não acolhe aspirações da União Européia, como são que se tenha em consideração na negociação o caráter multifuncional da agricultura da União Européia e os problemas derivados do desenvolvimento da biotecnologia, que deram lugar à polêmica sobre alimentos transgênicos. A OMC quer que a redução das ajudas à exportação se acompanhe de um compromisso para sua eliminação, assim como maiores reduções nos impostos aduaneiros de todos os produtos agrícolas. 20.- En el texto, se dice que los primeros borradores de la declaración : a) favorecen la inclusión de ayudas agrícolas a la exportación b) abogan por la abolición de las ayudas agrícolas a la exportación c) propulsan la supresión de auxilios agrícolas a la exportación d) propenden a la suspensión de las ayudas agrícolas a la exportación e) cuestionan la anulación de ayudas agrícolas a la exportación 21 En el texto, la palabra “borrador” significa www.pontodosconcursos.com.br 9
  • 10. CURSOS ON-LINE – ESPANHOL – CURSO REGULAR PROFESSOR GERARDO SANMARCO a) proyecto b) estudio c) encuesta d) minuta e) presupuesto 22.- En el texto se dice que a) La UE pretende incluir en la negociación la polémica sobre alimentos transgénicos b) La OMC defiende el estímulo al desarrollo biotecnológico c) La UE aspira a que se aumente la cantidad de productos agrícolas que circulen en el mercado común comunitario d) La OMC rechaza la restricción de subsidios que provoquen distorsiones en el comercio e) La OMC quiere que los productos entren en los países pagando tasas menores. GABARITO COMENTADO DA PROVA 2000 Questão 20 a) Falso. È justamente o oposto. b) A alternativa tem pouca diferença com a certa que é a “c” . A pequena diferença está no verbo “propulsam” (de ação) que é mais adequado que “abogan”(de argumentação) ou “propenden” (de inclinação). c) Verdadeira. d) Falsa. A palavra “propendem” (tem tendência) é muito tênue. Tampouco se fala de”suspensão” e sim de supressão (eliminação) e) Falsa. A palavra “cuestionam” é muito fraca para identificar a posição da OMC que é totalmente definida contra esse tipo de ajudas. Questão 21. a) Falsa. No caso como se trata de uma declaração e não um projeto, a palavra “borrador” não se ajusta. Se fosse algo anterior a um projeto seria “anteproyecto” b) Falsa. A palavra “estudo” não corresponde com rascunho. c) Falsa. “Encuesta” significa pesquisa. d) Verdadeira. Trata-se de um texto eliminar que será colocado em debate e) Falsa. “Presupuesto” significa orçamento. Questão 22 a) Falsa. O texto diz que a UE não foi contemplada com a inclusão desse assunto. www.pontodosconcursos.com.br 10
  • 11. CURSOS ON-LINE – ESPANHOL – CURSO REGULAR PROFESSOR GERARDO SANMARCO b) Falsa. O texto não cita posição da OMC sobre isso. c) Falsa. A UE é a principal opositora do livre comercio de produtos agrícolas. d) Falsa. É o contrário = A OMC rechaça os subsídios que provocam distorções no comércio. e) Verdadeira. A OMC quiere a retirada das restrições alfandegárias ao comércio internacional. SIMULADO 29 (Sobre TLC) Colombia firmó el TLC con EEUU Colombia y Estados Unidos firmaron este miércoles el Tratado de Libre Comercio (TLC) que contempla la reducción de aranceles y establece normas comunes para la protección a la propiedad intelectual y a los inversionistas, entre otras disposiciones. La firma llega después de 14 rondas de conversaciones durante 22 meses, negociaciones que Colombia empezó junto con Perú y Ecuador. "Ésta es una victoria simbólica para la democracia en Colombia", dijo el "número dos" en materia de comercio exterior de EEUU, John Veroneau, quien suscribió el acuerdo en nombre de su país. Por el lado colombiano, la firma fue del ministro de Comercio, Industria y Turismo, Jorge Humberto Botero, quien también resaltó el impacto del acuerdo trasciende el ámbito económico. "Este tratado es de gran importancia en el marco de la lucha denodada y sin cuartel que libra Colombia, con el decidido apoyo de Estados Unidos, contra el tráfico de drogas y el terrorismo", afirmó Botero en el acto, que tuvo lugar en el Banco Interamericano de Desarrollo (BID), en Washington. Perú ya firmó y ratificó su propio acuerdo, que precisa de la aprobación del Congreso de EEUU para su entrada en vigor, mientras que el proceso entre Quito y Washington está en suspenso. El TLC entre Colombia y EEUU se comenzará a aplicar una vez que logre el visto bueno de las legislaturas de ambos países, así como de la Corte Constitucional. Tras la ceremonia, Botero dijo esperar que el Congreso de EEUU pueda iniciar el debate sobre el TLC "lo antes posible". Para Colombia, la principal ventaja del acuerdo es un acceso "seguro" al mayor mercado del mundo, según destacó Veroneau. Actualmente, la mayoría de las exportaciones de Colombia entra www.pontodosconcursos.com.br 11
  • 12. CURSOS ON-LINE – ESPANHOL – CURSO REGULAR PROFESSOR GERARDO SANMARCO en EEUU sin pagar aranceles gracias a la llamada Ley de Preferencias Arancelarias Andinas y Erradicación de Drogas (ATPDEA). Pero este privilegio es temporal y depende de la renovación por el Congreso, como se pone de manifiesto actualmente, dado que la Ley expira en diciembre próximo. En cambio, para Estados Unidos el TLC eliminará las barreras a la participación de empresas de su país en el sector de los servicios en Colombia, protegerá la propiedad intelectual de sus farmacéuticas y otras compañías, y establecerá un mecanismo para la resolución de disputas de inversión, dijo Veroneau. Además, facilitará la entrada de las exportaciones de EEUU en un mercado de 44 millones de personas.1/12/2006 1.- En el texto se dice que el TLC a) establece acuerdos comerciales entre EEUU, Colombia, Ecuador y Perú. b) recorta derechos aduaneros y cobija derechos de inversores c) fue culminado tras 14 interrupciones de negociaciones d) dicta normas eliminando impuestos al comercio exterior e) ha sido confirmado por los Congresos de los países signatarios 2.- Segundo Botero, el acuerdo a) simboliza los avances de la democracia colombiana b) permitirá la victoria sobre el terrorismo y el tráfico de drogas c) refuerza la posición de Colombia en el Banco Interamericano de Desarrollo d) va más allá de lo meramente económico e) dará a Colombia un acceso al mayor país del mundo 3.- En el texto se dice que a) El congreso americano debatirá el TLC cuanto antes b) el TLC no modifica en nada los acuerdos comerciales ya existentes c) el TLC será temporal y sujeto a permanentes renovaciones del Congreso d) los productos colombianos nunca pagaron arancel para entrar en EEUU e) el TLC tiene varias etapas antes de entrar en vigor 4.- En el textos se dice que en el TLC a) se establecieron acuerdos iguales para las dos partes b) hay facilidades para las ventas externas de EEUU a Colombia c) se establece el acceso irrestricto de las empresas estadounidenses en Colombia. d) se establece un mercado de 44 millones como límite de las ventajas e) favorecerá más a EEUU de que a Colombia www.pontodosconcursos.com.br 12
  • 13. CURSOS ON-LINE – ESPANHOL – CURSO REGULAR PROFESSOR GERARDO SANMARCO 5.- La expresión “que logre el visto bueno” equivale en ese contexto a a) que siga con una buena visión b) que se debata poco rato c) que obtenga dictamen favorable d) que consiga ser soslayado e) que mitigue las contradiciones 6.- La expresión “en cambio” equivale en ese contexto a a) Tras la transformación b) Cuando cambie c) Dentro de la alteración d) Por el contrario e) En suma 7.- La expresión “además” equivale en ese contexto a a) asimismo b) por demás c) por ello d) en cuanto a e) sin embargo FICHA GRAMATICAL 14 SOBRE NEXOS INTERPARÁGRAFOS Eles são peças fundamentais para dar coesão a um texto. Eles estabelecem conexões de diversos tipos conforme o autor queira vincular as idéias e os argumentos enumerados. A seguir, veremos os principais , com sinônimos e traduções Asimismo = además = también. Indica que os parágrafos estão associados pela fonte ou pelo conteúdo. De hecho = en efecto = efectivamente. Indica que o parágrafo seguinte é confirmação do anterior, detalhado através de fatos ou exemplos práticos. Por ello = por lo tanto = de ahí que. Indica que o parágrafo seguinte é conseqüência do anterior. En cambio = por el contrario =por otra parte. Indica que o parágrafo seguinte é muito diferente do anterior no conteúdo informativo En suma = en resumen. Indica que o parágrafo seguinte é um resumo dos conceitos e/ou argumentos mencionados nos parágrafos anteriores. www.pontodosconcursos.com.br 13
  • 14. CURSOS ON-LINE – ESPANHOL – CURSO REGULAR PROFESSOR GERARDO SANMARCO Sin embargo = empero = no obstante. Indica que o parágrafo seguinte desenvolverá informações e/ou argumentações opostas às do parágrafo anterior. SIMULADO 30 (sobre MERCOSUR) Inmersos en un debate sobre los retos del mayor mecanismo de integración del área, los cancilleres de Argentina, Brasil, Uruguay y Paraguay, más Venezuela, celebran en Buenos Aires la V Reunión Extraordinaria del Consejo del Mercado Común (CMC). El encuentro del CMC, principal instancia de decisión del MERCOSUR, avanza en la incorporación de Caracas como socio pleno del bloque sureño, el tema que concentrará la atención de los mandatarios los días 20 y 21 de julio venideros. La cita, llamada por Argentina en su condición de presidente temporal de la unión aduanera, función que traspasará a Brasil en el segundo semestre, debate, además, una posición común con miras a la próxima reunión de la Organización Mundial de Comercio. Los ministros también evalúan las asimetrías que afectan el comercio dentro del MERCOSUR, en particular a Uruguay y Paraguay, las economías más débiles de este mecanismo de concertación. Según lo previsto, en el cónclave de mandatarios se suscribirán los protocolos de adhesión de la República Bolivariana al bloque sureño como su quinto miembro oficial. El ingreso de Venezuela, considerada la tercera economía de Sudamérica, es vista al interior de la asociación como un factor de fortalecimiento del proceso de integración real en el Cono Sur, pese a todas sus limitaciones y debilidades. Luego de su firma, esos documentos deberán ser sancionados por los parlamentos de Venezuela y de los cuatro socios fundadores de la unión aduanera. El borrador del pacto fija un plazo máximo de cuatro años a partir de su ratificación para que Caracas adopte el arancel externo común y el acervo normativo del MERCOSUR. Establece, además, plazos que llegan hasta 2013 para el libre comercio entre la República Bolivariana y el resto de los miembros de la unión aduanera sudamericana. Fuentes diplomáticas argentinas explicaron que en los protocolos se fija un cronograma de desgravación arancelaria por parte de la nación andina, el cual será de inmediata aplicación para Uruguay y Paraguay, los socios menores del bloque. Para el caso de Argentina y Brasil, las exenciones tendrán un carácter gradual y con plazos variables hasta el 2018, con lo cual se logró un reconocimiento explícito de las asimetrías comerciales dentro del MERCOSUR. www.pontodosconcursos.com.br 14
  • 15. CURSOS ON-LINE – ESPANHOL – CURSO REGULAR PROFESSOR GERARDO SANMARCO 1.- Según el texto, la reunión del Consejo del MERCOSUR en julio a) concluye el proceso de integración de Venezuela como quinto miembro b) decide quien asumirá la presidencia pre-tempore del MERCOSUR c) analiza los desequilibrios comerciales en el seno del bloque d) observa la situación de los países débiles que comercian con el MERCOSUR e) incluye pláticas para la incorporación a la OMC 2.- En el texto se dice en relación a la integración de Venezuela a) tendrá hasta 2010 para adecuarse al acervo de resoluciones del MERCOSUR b) tendrá que ser apoyada en los 5 suscritores por sendas mayorías parlamentares c) tendrá que implantar el arancel externo a humo de pajas. d) que serán pesadas sus debilidades y fuerzas para integrar el bloque e) que es la tercera mayor economía del continente americano 3.-En relación al borrador que será evaluado en la reunión del CM, él plantea que Venezuela a) adhiera al imposto común de importación externa al MERCOSUR en cuatro años b) implante de forma completa su arancel externo común con Uruguay y Paraguay hasta 2013 c) implante su arancel externo común con Brasil y Argentina en 12 años d) Venezuela conceda exenciones aduaneras irrestrictas a sus socios del MERCOSUR e) tiene cuatro años para iniciar la desgravación de aranceles con sus socios del MERCOSUR. 4.-El término “desgravación arancelaria” (li.23) se refiere específicamente a a) los acuerdos para reglamentación de la base imponible b) decisiones presupuestarias c) retiradas de las barreras impositivas y sanitarias a la importación d) rebaja gradual o total de tributos al comercio exterior e) exoneración de gravámenes a la actividad económica en los países miembros 5.- La expresión “pese a” (li.18) en ese contexto equivale a a) por causa de b) sin embargo de c) sino por d) mismo con www.pontodosconcursos.com.br 15
  • 16. CURSOS ON-LINE – ESPANHOL – CURSO REGULAR PROFESSOR GERARDO SANMARCO e) siempre y cuando 6. La expresión “luego de” (li.19) en el texto equivale a a) apués b) en seguida c) conque d) a menudo e) tras 7.-La expresión “los retos” (li.10) en el texto equivale a a) los acuerdos b) los hechos c) los desafíos d) los desencuentros e) tropiezos 8.- La expresión “con miras a” (li.9) significa a) precaviéndose de b) de ojo en c) soslayando d) sintonizándose con e) interfiriendo en SIMULADO 31 (sobre China) China es un socio “muy importante” para el desarrollo de América Latina, dijo hoy en Hong Kong el diplomático chileno Miguel Poklepovic. El chileno explicó que “el hecho es que China es muy importante para el desarrollo de América Latina, mientras que al contrario, eso no es tan obvio”. En la actualidad, China, fervientemente interesada en las materias primas que ofrece la región, ha encontrado en el nuevo continente un socio de primer nivel. Y no sólo eso, sino que la balanza comercial entre ambas partes ascendió a 400.000 millones de dólares en el año 2004, y fue especialmente importante en los casos de México, Brasil y Panamá. “No sólo el comercio entre China y América Latina seguirá aumentando, sino que casi que el 50% de las inversiones de China van a ir a parar allí”, dijo a Efe Poklepovic. Según fuentes oficiales, el 49.3% de las inversiones chinas a nivel mundial del año 2004 estuvieron destinadas a América Latina, por encima incluso de las que las empresas chinas dedicaron a Hong Kong. “Es cierto que ha habido un cambio en los intereses de China, pero eso se hubiera quedado en agua de borrajas si no hubiera sido recíproco por parte de América Latina”, aseguró. “Ahora (los chinos) poseen no sólo derechos de suministro, sino también minas, en Chile o en Perú”, dijo el diplomático. Desde que China alcanzó sus más altos niveles de desarrollo, en las últimas décadas, ha dirigido hacia América Latina, así como hacia el resto www.pontodosconcursos.com.br 16
  • 17. CURSOS ON-LINE – ESPANHOL – CURSO REGULAR PROFESSOR GERARDO SANMARCO del Tercer Mundo, una “diplomacia de los recursos”. No obstante, según los expertos, el creciente interés de los asiáticos por ese continente no representa una amenaza para Washington, quien ha dominado tradicionalmente la zona. Sin embargo, Poklepovic aseguró que el petróleo no es uno de esos recursos estratégicos, ya que las exportaciones del oro negro de América Latina a China se circunscriben a pequeños acuerdos con Venezuela, Cuba y Brasil. Sí lo son, por el contrario, alimentos como la soja procedentes de Brasil y de Argentina. En la actualidad, “Argentina y Brasil proporcionan el 29% de las importaciones totales de comida de China”. Además, China tiene también importantes intereses políticos en la zona. En la actualidad doce de los 26 países que todavía reconocen al gobierno de Taipei son latinoamericanos, lo que es un “factor irritante” para China a la hora de tratar con bloques comerciales como Mercosur, en el que se incluye Paraguay. 1.- En el texto se dice que a) en 2004, el intercambio comercial China/América Latina aumentó en 400 mil millones de dólares b) casi el 50% del comercio exterior chino está dirigido a América Latina c) América Latina es reacia al intercambio comercial con China d) los chinos controlan los recursos minerales de Chile y otros países e) el desplazamiento de capitales chinos al exterior tiene como meta principal A. Latina 2.- En el texto se dice que a) hay un entredicho diplomático entre China y Paraguay por causa de Taipei b) el MERCOSUR irrita a China por motivos económicos c) Argentina y Brasil son responsables por 30% del consumo alimentar de China d) el interés de China por A. Latina apunta a sus necesidades energéticas e) China realiza pequeñas importaciones de oro procedente de A. Latina 3.- En ese contexto, la expresión “derechos de suministro” (li.14) equivale a a) tasas de importación b) reservas de compra c) depósitos portuarios d) empresas comercializadoras e) estoques de reserva 4.- En ese contexto, la expresión “agua de borrajas” tiene el sentido de www.pontodosconcursos.com.br 17
  • 18. CURSOS ON-LINE – ESPANHOL – CURSO REGULAR PROFESSOR GERARDO SANMARCO a) tempestuoso b) sospechoso c) insostenible d) sin importancia e) inexplicable Lecturas Atualizadas sobre OMC, blocos e TLCs Texto 1 (Sobre el TLC Colombia EEUU) La situación textil y de la vestimenta podría agravarse aún más en los próximos meses, y tanto desde el sector industrial como desde el gobierno se buscan soluciones urgentes. Para el presidente de la Cámara Industrial de la Vestimenta, Elbio Fuscaldo, se está en un momento "bisagra" (dobradiça) y de transición, en un mercado de signado por una continua suba de los costos y las pérdidas de los mercados externos. Los factores principales que enfrenta la industria uruguaya en el contexto externo son la desaparición de la cuotificación que China tenía para exportar a Estados Unidos y ahora todas las empresas ponen su mirada en ese gigante asiático. La exportación ha caído a unos U$S 60 millones por año. A esto se suma un hecho (um fato) no menor: el acuerdo de Libre Comercio entre Estados Unidos y Colombia, que se ha convertido en la nueva cuna(berço) de la vestimenta en Latinoamérica, no sólo con una mano de obra barata, sino con tecnología, moda y diseño. Actualmente Colombia y Estados Unidos tienen un sistema de preferencias que será sustituido por el TLC. Pero ese sistema ya llevó a que la industria de la vestimenta uruguaya se alejara (afastasse) del principal consumidor del mundo. Mientras que Uruguay debe pagar un arancel de ingreso de 18%, Colombia no paga ni pagará aranceles. Es por esta razón que el ministro de Industria, Jorge Lepra, señaló en varias ocasiones que el sector textil sería el más afectado si no se firmaba un TLC con Estados Unidos, porque Colombia está a punto de convertirse en su principal competidor de ropa de calidad y a bajo precio. Porque si bien la industria textil exporta, gran parte de ella debe de vivir del mercado interno, el cual se ve saturado por ropa que ingresa desde Argentina y en especial desde China. - La subfacturación en el ingreso de mercadería y el contrabando también están afectando al sector.(larepublica.uy 25.11.2006) Texto 2 Bogotá. - El Parlamento Andino se declaró hoy preocupado por el Tratado de Libre Comercio (TLC) firmado por Colombia y Perú con Estados Unidos al advertir que en las discusiones para concretar www.pontodosconcursos.com.br 18
  • 19. CURSOS ON-LINE – ESPANHOL – CURSO REGULAR PROFESSOR GERARDO SANMARCO la alianza hubo "aspectos que pasaron desapercibidos" y por ello solicitó a los países andinos realizar un debate más profundo de la negociación. El presidente del Parlamento Andino, el senador colombiano Luís Fernando Duque, insistió en la necesidad de analizar minuciosamente los acuerdos realizados con el gobierno estadounidense para evitar que los tratados generen un desequilibrio en "el bienestar de nuestros pueblos y el crecimiento de nuestras economías", citó DPA. "Es indispensable que los congresos de Colombia y Perú escuchen a los representantes del órgano deliberante de la Comunidad Andina (CAN). Hemos adelantando una serie de análisis sobre los efectos del TLC en la normativa comercial y la institucionalidad del proceso andino de integración y tenemos mucho que aportar al debate al interior de los congresos sobre los efectos del TLC en la CAN", expresó Duque. Asimismo, recordó que en el texto actual del TLC negociado entre Bogotá y Washington no hay claridad sobre el tema agrícola que quedó sin salvaguardias, franjas de precios o productos excluidos "sin ser muy claro el acceso real al mercado de Estados Unidos en condiciones de competitividad". "El proceso andino de integración va mucho más allá de los aspectos comerciales, como no sucede con el TLC con Estados Unidos; la CAN se ha preocupado por promover y atender los aspectos sociales y culturales de los ciudadanos andinos, por esto, se debe analizar este Tratado Comercial a la luz de lo que se ha construido en 37 años de integración andina", agregó. Duque se declaró partidario de los acuerdos comerciales con terceros pero insistió en la necesidad de que estas alianzas comerciales "fortalezcan la capacidad competitiva de nuestros aparatos productivos y ayuden a mejorar el nivel de vida de los pueblos andinos". Los gobiernos de Estados Unidos y Colombia firmaron ayer en la sede del Banco Interamericano de Desarrollo en Washington un Tratado de Libre Comercio y quedaron a la espera de su aprobación en los respectivos congresos, donde se espera afrontará (enfrentará) los más duros cuestionamientos. Por su parte, el tratado con Perú, impulsado por el ex presidente Alejandro Toledo y apoyado por el del actual mandatario, Alan García, ya fue firmado por los ejecutivos de Lima y Washington y www.pontodosconcursos.com.br 19
  • 20. CURSOS ON-LINE – ESPANHOL – CURSO REGULAR PROFESSOR GERARDO SANMARCO refrendado por el Congreso peruano, pero falta que el Legislativo estadounidense lo apruebe. Texto 3 Negociaciones sobre subsídios bloqueadas GINEBRA.- El Grupo de los Veinte (G-20), que reúne a países en desarrollo, anunció hoy que responderá a las iniciativas de la UE y EEUU para desbloquear las negociaciones agrícolas de la Organización Mundial de Comercio (OMC) con una "propuesta propia y con números concretos" que se referirá de forma concreta a los tres pilares de la negociación agrícola, como son los subsidios a la exportación, el apoyo interno y el acceso a los mercados", dijo el ministro brasileño Celso Amorim. "En apoyo interno (a los agricultores) buscamos medidas que supongan un auténtico recorte de esos niveles, y en acceso a mercados, verdaderas oportunidades para acceder a ellos, con recortes como los que hubo en la Ronda de Uruguay", precisó Amorim. Agregó que para el G-20 es esencial que "en 2010 se eliminen todas las formas de apoyo a las exportaciones y las que se dan a nivel nacional" en la agricultura. Las diferencias en agricultura, uno de los capítulos claves de esta negociación, bloquean el resto de los acuerdos sobre servicios, acceso a mercados para productos industrializados o facilitación comercial, entre otros. El Representante Especial de Comercio de EEUU, Rob Portman, presentó este lunes una iniciativa que afecta a los tres pilares agrícolas y que se ejecutaría en dos etapas, una inicial en la que aplicarían recortes significativos en los aranceles y en las ayudas directas en un periodo de cinco años. Cinco años después se pasaría a otra fase, en la que se procedería a la total eliminación de las políticas que distorsionan el comercio agrícola. La UE, por su parte, propuso reducir un 70% las ayudas agrícolas internas que distorsionan el comercio, las que en el argot de la OMC se consideran de la "caja azul" o de la "caja amarilla", es decir que distorsionan el comercio y que están ligadas a la producción o a los precios. Asimismo, está dispuesta a rebajar el 65% los llamados apoyos que dentro de la OMC están exentos de rebajas siempre y cuando no superen el 5% del valor total de la producción agrícola. Texto 4 GINEBRA.- Las potencias comerciales del mundo hicieron avances menudos (pequenos)en las últimas décadas en las iniciativas para reducir los subsidios agrícolas que distorsionan el comercio, de acuerdo con un estudio publicado el martes por la www.pontodosconcursos.com.br 20
  • 21. CURSOS ON-LINE – ESPANHOL – CURSO REGULAR PROFESSOR GERARDO SANMARCO OCDE. El informe de la OCDE -que agrupa a las naciones más desarrolladas y tiene su sede en París- mostró también que Islandia, Suiza y Noruega mantenían los niveles más altos de apoyo a los agricultores entre los 30 miembros de la OCDE. Además, Estados Unidos incrementó sus niveles de protección en el 2004. "Las reformas de las políticas se han centrado en cambiar la forma en la que se ofrece la ayuda a los productores, con un alejamiento (afastamento)notable de las medidas relacionadas con el volumen de producción", dijo el reporte en referencia a los pagos que reciben los granjeros y que están atados a su producción. No obstante, aunque estas reformas podrían continuar en los próximos años, dijo la OCDE, la participación de esas formas de respaldo que tanto distorsionan la producción y el comercio igualmente representó el 74% de todos los subsidios en el período 2002- 2004. Esto reflejó una reducción desde el 91% del lapso 1986- 1988. El apoyo agrícola de la UE surgió como un tema importante el fin de semana pasado cuando fracasó un encuentro de los líderes del bloque para ponerse de acuerdo sobre el presupuesto de la Unión Europeia para el período 2007-2013. La falta de entendimiento en gran medida se debió a que Francia rechazó el pedido de Gran Bretaña para que se recorten los subsidios al agro.Esta cuestión también es uno de los ejes(eixos) de la Ronda de Doha de negociaciones para la liberalización comercial, un proceso en el seno de la Organización Mundial del Comercio (OMC) que ha avanzado despacio(devagar). (emol- 22/6/05) Texto 5 (Conferencia OMC- 2005) Los países ricos han roto(quebrado), durante la preparación a la Cumbre de la OMC que se celebra la semana que viene en Hong Kong, las promesas que hicieron hace cuatro años. El informe de la ONG Intermón Oxfam muestra cómo al pedir más y más a los países pobres, a cambio de muy poco, le han dado la espalda (as costas) a lo que tenía que haber sido la 'Ronda de Desarrollo'. "Las negociaciones, que deberían haber llevado a una reforma de las reglas comerciales para potenciar el desarrollo, han descarrilado(derrapado) por culpa de los tiras y aflojas de los más ricos afirma Gonzalo Fanjul, coordinador de Intermón Oxfam. "Los países más desarrollados exigen un gran número de concesiones, porque dicen que necesitan ver 'sangre en el suelo' de todos para poder vender el acuerdo en la OMC en sus propios países. Esta posición de poder le ha dado la vuelta a las promesas hechas (feitas) en Doha. Con la tendencia actual, las negociaciones tienen poco que ofrecer a los que más lo necesitan", añade Fanjul. Fanjul denuncia que la reforma en la agricultura es esencial para millones de personas, pero los países más ricos no han dado las www.pontodosconcursos.com.br 21
  • 22. CURSOS ON-LINE – ESPANHOL – CURSO REGULAR PROFESSOR GERARDO SANMARCO pasos para acabar con los subsidios a la exportación ni para mejorar el acceso a sus mercados. Mientras tanto, los Estados más pobres reciben presiones para liberalizar sus mercados. Las negociaciones sobre los aranceles industriales son, si cabe, más preocupantes. El borrador acordado en julio de 2004 puede destruir los sectores industriales de muchos países pobres. Además, les están obligando a recortar sus aranceles por encima de los que los países ricos pretenden realizar, lo que representa una contradicción de las promesas hechas en Doha. Respecto a los servicios, en lugar de respetar el método de inclusión acordado en Doha, los países ricos están pidiendo la aplicación de un compromiso mínimo respecto al número de sectores aceptados y el nivel de apertura, que podría llevar a una liberalización forzada de sectores sociales básicos de los países más pobres. Si el nivel de intransigencia se mantiene, las negociaciones pueden quedar colapsadas a lo largo de la próxima década. Por desgracia, las alternativas a un acuerdo fuera de la OMC tampoco son atractivas. Las negociaciones comerciales regionales o bilaterales ponen en gran riesgo las promesas de desarrollo. Texto 6 (Sobre TLC Perú-EEUU) Perú y Estados Unidos cerraron ayer con un acuerdo sus negociaciones para un Tratado de Libre Comercio, el primero en dos años con un país suramericano después de Chile.. El acuerdo fue logrado bilateralmente, pese a que Perú inició el proceso de negociación en bloque con Ecuador y Colombia. Se desconoce de momento si con ello se rompería definitivamente el frente andino que conformaron esos tres países. Fuentes de la delegación peruana dijeron que no formularían ningún comentario sobre el acuerdo antes de que fuera anunciado en Lima por el presidente Alejandro Toledo. El presidente George W. Bush deberá notificar al Congreso con 90 días de anticipación su “intención de firmar” el tratado con Perú, un procedimiento que es rutinario. Pero el envío al Congreso para ratificación puede complicarse debido a que el próximo año en noviembre habrá elecciones legislativas y pocos legisladores desean entrar en un tema que ha tenido tenaces discrepantes. Estados Unidos ha firmado ya acuerdos de libre comercio con México, Chile y cinco países centroamericanos más República Dominicana agrupados en el convenio llamado CAFTA.. Está adicionalmente negociando con Panamá. Colombia y Ecuador se retiraron de las negociaciones el 22 de noviembre en Washington luego de 19 meses de tratos iniciados en la ciudad colombiana de Cartagena. Los dos países dijeron que www.pontodosconcursos.com.br 22
  • 23. CURSOS ON-LINE – ESPANHOL – CURSO REGULAR PROFESSOR GERARDO SANMARCO no podían seguir hablando mientras Estados Unidos se mostraba intransigente en sus peticiones. Perú, en cambio, pidió continuar las negociaciones en la primera semana de diciembre, lo cual hizo a partir del lunes en que una delegación de varios ministros llegó a Washington para negociar prácticamente 24 horas al día. En su intento de avanzar, el presidente Alejandro Toledo instruyó a sus negociadores a que terminen el proceso a como diera lugar, dijeron fuentes informadas. Los ministros de Agricultura, Comercio Exterior, Producción y Salud, que integran el frente negociador peruano, se habían estado resistiendo a algunas propuestas estadounidenses, ante lo cual Toledo envió a su hombre de confianza, su primer ministro y presidente del consejo de ministros Pedro Pablo Kuczynski, para facilitar un entendimiento. 5/12/2005 Texto 7 (Sobre conflitos de comércio exterior) BRUSELAS.- La Unión Europea ha vuelto a fracasar en su intento de buscar un consenso sobre los aranceles al calzado de cuero importado desde China y Vietnam. Los 25 tendrán una última oportunidad el próximo miércoles, dos días antes de expire el actual régimen de tarifas temporales. Los representantes de los Estados miembros trataron de alcanzar un acuerdo en base a la última propuesta de la Comisión, que establece un arancel durante tres años de 16,5% para el calzado de China y del 10% para el importado de Vietnam. Éste es el último planteamiento de Bruselas para sustituir al sistema temporal de aranceles progresivos que empezó a aplicarse el pasado mes de abril y que en la actualidad alcanza el 19,4% para los zapatos de China y el 16,8% para los de Vietnam. Este mecanismo, que concluirá el 6 de octubre, dará lugar a la total liberalización de las importaciones si no se halla un acuerdo alternativo. Estos aranceles progresivos y temporales hacen referencia a medidas 'antidumping' que se impusieron después de que una investigación de la Comisión Europea concluyera que existe una fuerte intervención del Estado en los dos países asiáticos que provoca que los zapatos se vendan en Europa por debajo de su coste de producción. Los Estados miembros llevan negociando desde hace meses qué medidas de 'antidumping' podrían aplicarse a partir del 7 de octubre, pues los 25 se debaten entre dos posiciones: una, la de los países productores que, como España o Italia, apuestan por la imposición de medidas y ven con buenos ojos la última propuesta de la Comisión, y dos, la de los países con un potente sector distribuidor, particularmente los nórdicos, que apuestan por la liberalización. www.pontodosconcursos.com.br 23
  • 24. CURSOS ON-LINE – ESPANHOL – CURSO REGULAR PROFESSOR GERARDO SANMARCO No obstante, fuentes españolas explicaron que Chipre se ha sumado al grupo de países productores junto a España, Francia, Italia, Portugal, Polonia, Lituania y Hungría. La decisión se tomará por mayoría simple, de manera que, con la suma de Chipre, solamente haría falta un Estado miembro que votara con el grupo del que forma parte España o que se abstuviera para llegar al acuerdo. A este respecto, aventuraron fuentes españolas que "quizás" Austria o Letonia podrían cambiar su postura. Francia, además, formuló una nueva propuesta en aras del nuevo compromiso, que consiste en que el nuevo régimen de aranceles se aplique durante dos años en lugar de tres como proponía la Comisión. El portavoz de Comercio del Ejecutivo comunitario, Peter Power, mantuvo la cautela sobre esta nueva propuesta, que primero tendrá que ser estudiada por el comisario del ramo, Peter Mandelson, y posteriormente ser aprobada por los 25 comisarios. "Es un periodo un poco corto pero si los Estados miembros quieren, entonces tendremos que estar con ellos". GABARITO SIMULADO 30 1.- En el texto se dice que el TLC b) recorta derechos aduaneros (aranceles) y cobija (protege)derechos de inversores 2.- Segundo Botero, el acuerdo d) va más allá de lo meramente económico 3.- En el texto se dice que e) el TLC tiene varias etapas antes de entrar en vigor 4.- En el textos se dice que en el TLC b) hay facilidades para las ventas externas de EEUU a Colombia 5.- La expresión “que logre el visto bueno” equivale en ese contexto a c) que obtenga dictamen favorable 6.- La expresión “en cambio” equivale en ese contexto a d) Por el contrario 7.- La expresión “además” equivale en ese contexto a a) asimismo GABARITO SIMULADO 30 www.pontodosconcursos.com.br 24
  • 25. CURSOS ON-LINE – ESPANHOL – CURSO REGULAR PROFESSOR GERARDO SANMARCO 1.- Según el texto, la reunión del Consejo del MERCOSUR en julio c) analiza los desequilibrios comerciales(ASIMETRÍAS)en el seno del bloque 2.- En el texto se dice en relación a la integración de Venezuela b) tendrá que ser apoyada en los 5 suscritores (V4ENEZUELA, BRASIL, Argentina, Paraguay, Uruguay) por sendas mayorías parlamentares. 3.-En relación al borrador que será evaluado en la reunión del CM, él plantea que Venezuela a) adhiera al imposto común de importación externa al MERCOSUR en menos de cuatro años 4.-El término “desgravación arancelaria” (li.23) se refiere específicamente a d) rebaja gradual o total de tributos al comercio exterior 5.- La expresión “pese a” (li.18) en ese contexto equivale a d) mismo con 6. La expresión “luego de” (li.19) en el texto equivale a e) tras 7.-La expresión “los retos” (li.10) en el texto equivale a c) los desafíos 8.- La expresión “con miras a” (li.9) significa b) de ojo en GABARITO SIMULADO 31 1.- En el texto se dice que e) el desplazamiento(deslocamento) de capitales chinos al exterior tiene como meta principal A. Latina 2.- En el texto se dice que a) hay un entredicho diplomático entre China y Paraguay por causa de Taipei 3.- En ese contexto, la expresión “derechos de suministro” (li.14) equivale a b) reservas de compra 4.- En ese contexto, la expresión “agua de borrajas” tiene el sentido de d) sin importancia www.pontodosconcursos.com.br 25